Site Loader

Содержание

Напряжение эдс формула. Внутреннее сопротивление — источник

Внутреннее сопротивление источников тока пренебрежимо мало.
Внутреннее сопротивление источника тока пренебрежимо мало.
Внутреннее сопротивление источника тока, рассчитанное по данной формуле, будет, строго говоря, действительно только для данного интервала нагрузок вследствие того, что поляризация не пропорциональна плотности тока.
Внутреннее сопротивление источника тока — сопротивление, которым обладает, источник тока. Это важная характеристика всякого источника тока, определяющая его внутреннее падение напряжения, напряжение, которое может создать источник на концах питаемой им цепи, и тот наибольший ток, который может дать источник при коротком замыкании.
Внутреннее сопротивление источника тока — сопротивление, которым обладает источник тока.
Внутренним сопротивлением источника тока, сопротивлениями соединительных проводов и контактов в ключах пренебречь.
Чему равно внутреннее сопротивление источника тока, ЭДС которого равна 30 В, если после включения внешней цепи сопротивлением 6 Ом напряжение на зажимах батареи стало равным 18 В.

Отсюда находим внутреннее сопротивление источника тока.
Здесь и далее внутренним сопротивлением источника тока и подводящих проводов следует пренебречь, если оно не задано в условии.
Здесь тэар при небольшом внутреннем сопротивлении источника тока и соответственно небольшом сопротивлении лампы rgK относительно невелико. Соответственно тзар, определяющееся в основном высоким сопротивлением RgK (получающимся в результате того, что при разряде потенциал сетки оказывается под отрицательным потенциалом относительно катода), становится во много раз больше, чем тзар и длина экспоненциального импульса на выходе (считая продолжительность для половины амплитуды) в несколько десятков раз превышает длительность импульса, по-данного на вход.
Определить электродвижущую силу и внутреннее сопротивление источника тока, если при одном положении движка реостата амперметр показывает 0 2 А, вольтметр — 1 8 В, а при другом положении движка — 0 4 Аи 1 6 В соответственно.
Обозначим через г — внутреннее сопротивление источника тока, через R — сопротивление каждого из вольтметров.
Ничем, так как внутреннее сопротивление источника тока бесконечно велико.
Сначала определим ЭДС и внутреннее сопротивление источника тока.

Напряжение на клеммах = напряжение без нагрузки — внутреннее сопротивление сила тока. Резюме: если источник тока или напряжения не загружен потребителем, ток не течет, и поэтому нет падения напряжения. На контактах источника питания можно измерять напряжение разомкнутой цепи. Когда к источнику питания подключена нагрузка, ток течет, а исходное напряжение разомкнутой цепи разделяется между сопротивлением нагрузки и внутренним сопротивлением источника питания.

Ток короткого замыкания: Наконец, существует третья возможность короткого замыкания контактов источника тока или напряжения. В то же время максимальный ток, который источник может обеспечить потоками. Он ограничен сопротивлением линии и внутренним сопротивлением источника напряжения. Важно: ток короткого замыкания, но напряжение короткого замыкания не существует теоретически. Существует ток короткого замыкания, который по определению является током, который течет, когда ток или источник напряжения закорочены.

Для определения ЭДС и внутреннего сопротивления источника тока к его выходу был подключен сначала резистор сопротивлением Д 2 Ом, затем — резистор сопротивлением Л2 4 Ом.
Наклон этих кривых определяется внутренним сопротивлением источника тока. В это понятие включается обычно как собственно омическое сопротивление, так и сопротивление, обусловленное поляризацией.
Здесь пренебрегают сопротивлением соединительных проводников и внутренним сопротивлением источника тока.
Для создания такого режима необходимо, чтобы внутреннее сопротивление источника тока было больше сопротивления базо-эмиг-терного перехода как в открытом, так и в закрытом состоянии. Чаще всего это условие выполняется при включении последовательно входу транзнсюра индуктивной катушки, которая одновременно является контурной катушкой.

При прохождении тока часть мощности выделяется на внутреннем сопротивлении источника тока.
К каким отрицательным последствиям приводит то, что внутреннее сопротивление источника тока дифференциального каскада имеет конечное значение.
Цепь из двух параллельных ветвей. U (в в течение t сек. равна.| Соотношения между единицами энергии. Мощность, передаваемая нагрузке, будет максимальной при раввщ ве внутреннего сопротивления источника тока и сопротивления нагрузки.
Зачастую серьезные недоразумения возникают у учащихся из-за неумения правильно учитывать влияние внутреннего сопротивления источников тока на режим работы всей электрической цепи. Ряд задач параграфа (например, 383, 385, 386, 392 — 395 и др.) посвящен специально выяснению этого вопроса, а также выяснению вопроса о выборе наиболее выгодных условий работы источников тока.
Кристаллы аммиаката цинка не-электропроводны, и образование этого соединения приводит к увеличению внутреннего сопротивления источника тока.
В любом замкнутом контуре (например, а ] 6, с алгебраическая сумма электродвижущих сил равна алгебраической сумме произведений величин токов на сопротивления отдельных участков цепи. Вычисляя сумму произведений токов на сопротивления отдельных участков цепи, следует учитывать также и внутренние сопротивления источников тока.
Если предположить, что емкость C0z пренебрежимо мала или включить ее в схему четырехполюсника Q, то внутреннее сопротивление источника тока / g можно считать действительным и равным У.
Получили, что максимальная мощность выделяется на нагрузке при условии, что величина внешнего сопротивления цепи R равна внутреннему сопротивлению источника тока.
Здесь под R понимается сопротивление всех резисторов, образующих цепь (сопротивление нагрузки), а под г — внутреннее сопротивление источника тока.

Существует также напряжение без нагрузки, которое по определению накладывается при отсутствии тока. Цель этой работы состояла в том, чтобы экспериментально определить электродвижущую силу, значение внутреннего сопротивления источника напряжения, используя в качестве параметра вариации восемь резисторов и проверить теорему максимальной передачи мощности от источника к резистивной нагрузке. При проверке условия максимальной передачи мощности наблюдалось, что это явление имело место, когда значения сопротивлений соответствовали, хотя даже с максимальной мощностью эффективность использования источника составляла всего 50%, поскольку половина генерируемой мощности рассеивается как тепло непосредственно в самом источнике.

Здесь под R понимается сопротивление всех резисторов, образующих цепь (сопротивление нагрузки), а под т — внутреннее сопротивление источника тока.

Механическая система и ее электрические модели (метод четырехполюсников. Как уже указывалось выше, внутреннее сопротивление источника напряжения (первая система аналогий) должно быть весьма малым, а внутреннее сопротивление источника тока (вторая система аналогий) — весьма большим, по сравнению с сопротивлением модели.
К положительным качествам рассматриваемого преобразователя следует отнести то, что в нем не предъявляется особо жестких требований к переходному сопротивлению ключей, так как величина их переходного сопротивления составляет лишь незначительную часть внутреннего сопротивления источника тока и не оказывает влияния на точность преобразования.
Итак, при последовательном включении п одинаковых источников тока электродвижущая сила образующейся батареи в п раз превышает электродвижущую силу отдельного источника тока, однако в этом случае складываются не только электродвижущие силы, но также и внутренние сопротивления источников тока. Такое включение является выгодным, когда внешнее сопротивление цепи весьма велико в сравнении с внутренним сопротивлением.
Следует отметить, что схема рис. 1 — 2 6 эквивалентна схеме рис. 1 — 1 а только в отношений энергии, выделяющейся в сопротивлении нагрузки R, и не эквивалентна ей в отношении энергии, выделяющейся во внутреннем сопротивлении источника тока.
Но Сумма разностей потенциалов замкнутой цепи равна нулю, сумма сопротивлений всех участков замкнутой цепи — это ее суммарное сопротивление, которое обычно записывают в виде двух слагаемых: R — внешнее (по отношению к источникам) сопротивление иг — внутреннее сопротивление источников тока.
Второй [ IMAGE ] Схема к примеру. В этом уравнении г и г % — внутренние сопротивления источни-ков тока е и е2 — на схеме не показаны; IR, IR2 и IRS — падения напряжения на внешних сопротивлениях цепи; / г, и / г2 — падения напряжений на внутренних сопротивлениях источников тока.
Внутреннее сопротивление источника тока может быть как чисто активным, так и реактивным.
Зависимость р / ро — отношения (выраженного в децибелах звукового давления на поверхности жесткого цилиндра (с высотой, равной его диаметру, куба, сферы к звуковому давлению, имевшему место в поле до их внесения, от отношения dA (или. /. — диаметра цилиндра или сферы (или ребра куба к длине волны. Параметр семейства кривых — угол Ф между осью цилиндра, куба, сферы и направлением прихода звука. При расчете микрофонных усилителей исходят из следующих соображений. Номинальное сопротивление микрофона является внутренним сопротивлением источника тока на входе усилителя, входное сопротивление усилителя — сопротивлением нагрузки микрофона.
В качестве источников тока в потенциометрии чаще всего применяют аккумуляторы или сухие элементы, значительно реже — стабилизированные источники постоянного тока. Современные потенциометры устроены таким образом, что внутреннее сопротивление источника тока не отражается на работе потенциометра. При работе с сухими батареями и аккумуляторами необходимо учитывать зависимость разрядного тока от времени, которая имеет минимальную крутизну через 10 — 15 мин после включения.
Распределение электрического напряжения вдоль обмотки сверхпроводящего магнита при образовании в нем нормальной зоны. На самом деле (рис. 9.2) высокий потенциал развивается внутри обмотки, где существует активная компонента напряжения, направленная навстречу индуктивной. Небольшая разность потенциалов между подводящими проводами обусловлена внутренним сопротивлением источника тока, который обычно автоматически отключается при переходе магнита в нормальное состояние. Но даже если это не произойдет, напряжение на источнике тока будет составлять всего лишь несколько вольт по сравнению с сотнями и, возможно, тысячами вольт в нормальной зоне. Поэтому напряжением источника можно пренебречь, но источник тока следует по возможности быстро отключить, чтобы не допустить длительного тепловыделения в обмотке и криостате.

Однако было отмечено, что результаты были удовлетворительными. Ключевые слова: резистор, сопротивление, электродвижущая сила, мощность, закон Ом, электроэнергия. Когда мы работаем с электрическими цепями, в некоторых случаях нам нужно устройство, которое поддерживает разность потенциалов между двумя терминалами, потому что, если бы не было этой разности потенциалов, не было бы электрического тока, протекающего по цепи, Для этого типа устройства он называется источником напряжения. Источником напряжения является устройство, которое при подключении к цепи подвергает носители заряда разности потенциалов, то есть обеспечивает энергию для движения через работу, выполняемую на носителях заряда.

Символом Rt на рис. 5.12, а обозначено внутреннее сопротивление источника тока.
Ключ, закорачивающий точку А на землю, с малым сопротивлением в открытом состоянии. Сопротивление открытого ключа обычно пренебрежимо мало по срав-нению с внутренним сопротивлением источника тока. Поэтому падение напряжения на ключе вызывает ничтожную погрешность.
Зависимость зарядного тока гео. На рис. 3 показана зависимость зарядного тока геометрической емкости от времени без учета токов абсорбции. Необходимо отметить, что спад тока в этом случае определяется внутренним сопротивлением источника тока, а не состоянием изоляции.
Хорошо, что при решении задачи Вы воспользовались методом эквивалентного активного двухполюсника. К сожалению, Вы ошиблись в определении значения сопротивления активного двухполюсника R3K: внутреннее сопротивление источника тока бесконечно велико, поэтому пассивный двухполюсник, к которому преобразуется схема рис. 6.13 а, при определении R3K будет содержать два резис-тивных элемента, соединенных последовательно.
К, так как в противном случае в выражении (5.1) должно быть учтено также напряжение непосредственно на входе усилителя. Вторым ограничивающим условием при выводе соотношения (5.1) является предположение о том, что внутреннее сопротивление источника тока весьма мало.
Таким образом, трансформатор изменяет величину сопротивления R в k2 раз. Этим широко пользуются при разработке различных электрических схем для согласования сопротивлений нагрузки с внутренним сопротивлением источников тока.
Основные типы однофазных трансформаторов.| Однофазные трансформаторы большой мощности. Таким образом, трансформатор изменяет величину сопротивления г в k2 раз. Этим свойством широко пользуются ьри разработке различных электрических схем для согласования сопротивлений нагрузки с внутренним сопротивлением источников тока.
Простей-шая электрическая цепь. Закон Ома справедлив не только для участка, но и для всей электрической цепи. В этом случае в значение R подставляется суммарное сопротивление всех элементов цепи, в том числе и внутреннее сопротивление источника тока. Однако при простейших расчетах цепей обычно пренебрегают сопротивлением соединительных проводников и внутренним сопротивлением источника тока.
Цепь постоянного тока. Напряжение, действующее во внешней электрической цепи источника тока, может быть представлено в виде суммы падений напряжения на отдельных элементах этой цепи. Но ведь ток, циркулирующий в цепи, протекает и через источник тока, который имеет свое сопротивление, называемое внутренним сопротивлением источника тока.

Именно факт выполнения работ на несущих нагрузках поддерживает разность потенциалов между терминалами. Эта «энергия», создаваемая источником напряжения, называется электродвижущей силой, также известной как ЭДС. Электродвижущая сила источника напряжения определяется как работа, выполняемая источником для передачи нагрузок от терминала нижней единицы заряда.

Источник напряжения может быть охарактеризован двумя типами: идеальным источником напряжения и фактическим источником напряжения. Идеальный источник напряжения — это тот, который не имеет сопротивления внутреннему перемещению нагрузок с одного терминала на другой. Разность потенциалов между выводами идеального источника равна его электродвижущей силе. В реальном источнике напряжения это не происходит. Внутри источника имеется несколько проводящих материалов, где каждый из них создает определенное сопротивление внутреннему перемещению нагрузок.

Допустим, есть простейшая электрическая замкнутая цепь, включающая в себя источник тока, например генератор, гальванический элемент или аккумулятор, и резистор, обладающий сопротивлением R. Поскольку ток в цепи нигде не прерывается, то и внутри источника он течет.

В такой ситуации можно сказать, что любой источник обладает некоторым внутренним сопротивлением, препятствующим току. Это внутреннее сопротивление характеризует источник тока и обозначается буквой r. Для или аккумулятора внутреннее сопротивление — это сопротивление раствора электролита и электродов, для генератора — сопротивление обмоток статора и т. д.

Рассматривая сопротивление, исходящее от всех проводников, встроенных в источник, мы можем определить их как единый резистор, эквивалентное сопротивление которого равно сумме сопротивлений всех проводников. Идеализация реального источника напряжения будет рассматривать его как идеальный источник напряжения, но с внутренним сопротивлением. Поэтому, когда реальный источник не подключен к цепи, разность потенциалов между выводами этого в точности равна значению его электродвижущей силы. С момента, когда источник подключен к цепи, он проводит ток, создавая разность потенциалов на выводах меньше, чем его электродвижущая сила.

Таким образом, источник тока характеризуется как величиной ЭДС, так и величиной собственного внутреннего сопротивления r – обе эти характеристики свидетельствуют о качестве источника.

Электростатические высоковольтные генераторы (как генератор Ван де Граафа или генератор Уимшурста), к примеру, отличаются огромной ЭДС измеряемой миллионами вольт, при этом их внутреннее сопротивление измеряется сотнями мегаом, потому они и непригодны для получения больших токов.

Через источник напряжения, а работа, выполняемая источником на зарядке, определяется уравнением, т.е. Согласно закону сохранения энергии работа, выполняемая источником, равна тепловой энергии, рассеиваемой в резисторах. Манипулируя уравнением, имеем. Разделив обе части уравнения на.

Тем не менее, разделив оба члена на. Целью эксперимента является определение значения внутреннего сопротивления источника напряжения и его электродвижущей силы. Для этого мы применяем метод, называемый методом сохранения энергии, чтобы вывести соотношение между проходящим через цепь электрическим током, электродвижущей силой источника и внутренними и внешними сопротивлениями.

Гальванические элементы (такие как батарейка) — напротив — имеют ЭДС порядка 1 вольта, хотя внутреннее сопротивление у них порядка долей или максимум — десятка Ом, и от гальванических элементов поэтому можно получать токи в единицы и десятки ампер.

Уравнение дает нам желаемое соотношение между. Тогда в реальном источнике разность потенциалов. Между двумя терминалами никогда не будет равняться силе. Электромотор, потому что, когда ток проходит через резисторы, система теряет потенциал. Таким образом, чем больше резистивных нагрузок в цепи, тем больше.

Учитывая взаимосвязь, предлагается определить. Мы манипулируем уравнением, чтобы оставить внешнее сопротивление в функции других членов. Тогда, поскольку уравнение прямой задается формулой. Рисунок 2 — Схема, используемая в эксперименте. В положении А в амперметре помещался мультиметр.

На данной схеме показан реальный источник с присоединенной нагрузкой. Здесь обозначены , его внутреннее сопротивление, а также сопротивление нагрузки. Согласно , ток в данной цепи будет равен:

Поскольку участок внешней цепи однороден, то из закона Ома можно найти напряжение на нагрузке:

Мы выбрали этот момент, потому что мы хотели бы определить падение напряжения в цепи в целом за счет изменения внешнего резистора. Итак, закрывая переключатель, измеряя. Обладая всеми упомянутыми материалами, схема была установлена ​​в соответствии с рисунком. Через показания, показанные в мультиметрах, значения напряжения и тока, связанные с используемым внешним резистором. Измерения проводились для восьми резисторов и, как правило, была построена таблица, связывающая падение напряжения и ток с каждым резистором.

Эти значения были отмечены и перечислены в таблице. Затем, по отношению к внешнему сопротивлению, падение напряжения и электрический ток, проходящие через резисторы, обратно пропорциональны. Необходимость показать эту связь между током и напряжением была только для изучения того, как эти величины ведут себя по мере изменения внешнего резистора. Таблица 1 — Связь между внешним сопротивлением, током и напряжением.

Выразив из первого уравнения сопротивление нагрузки, и подставив его значение во второе уравнение, получим зависимость напряжения на нагрузке от тока в замкнутой цепи:

В замкнутом контуре ЭДС равна сумме падений напряжений на элементах внешней цепи и на внутреннем сопротивлении самого источника. Зависимость напряжения на нагрузке от тока нагрузки в идеальном случае линейна.

Со значениями таблицы график был составлен, что связывает поведение падения напряжения в резисторах в зависимости от электрического тока, проходящего через схему. График 1 — Связь между напряжением и током. График 2 — Связь между падением напряжения и внешним сопротивлением.

Можно отметить, что это соотношение между падением напряжения и током в цепи обратно пропорционально. Таким образом, заменяя в, получаем. На графике можно найти экспериментальное определение значений электродвижущей силы и сопротивления. При этом необходимо было линеаризовать его, чтобы облегчить его анализ и интерпретацию. Для линеаризации была создана таблица, связывающая резисторы с обратным потоком, проходящим через них.

График это показывает, но экспериментальные данные на реальном резисторе (крестики возле графика) всегда отличаются от идеала:

Эксперименты и логика показывают, что при нулевом токе нагрузки напряжение на внешней цепи равно ЭДС источника, а при нулевом напряжении на нагрузке ток в цепи равен . Это свойство реальных цепей помогает экспериментально находить ЭДС и внутреннее сопротивление реальных источников.

Влияние внутреннего сопротивления на свойства двухполюсника

Эти значения были записаны в таблице. Электродвижущая сила. Таблица 4 — Сравнение значений, измеренных с экспериментальными значениями. Таблица 2 — Связь между внешним сопротивлением и обратным току. Кроме того, удалось проанализировать второй пункт, который должен был проверить теорему о состоянии передачи максимальной мощности от источника к резистивной нагрузке. Для этого был создан набор данных, связанный с мощностью, рассеиваемой каждым используемым резистором.

Со значениями таблицы график был нанесен на график, который показывает, по-видимому, линейное поведение сопротивления как функцию обратного тока. График 4 — Связь между внешним сопротивлением и обратным току. Основываясь на поведении графика, метод наименьших квадратов применялся для определения наилучшей линии, которая соответствует полученной точечной кривой. После применения метода и с помощью уравнений,, и, можно было определить значение электродвижущей силы и внутреннего сопротивления источника.

Экспериментальное нахождение внутреннего сопротивления

Чтобы экспериментально определить данные характеристики, строят график зависимости напряжения на нагрузке от величины тока, затем экстраполируют его до пересечения с осями.

В точке пересечения графика с остью напряжения находится значение ЭДС источника, а в точке пересечения с осью тока находится величина тока короткого замыкания. В итоге внутреннее сопротивление находится по формуле:

Для расчета мощности, рассеиваемой в каждом резисторе, использовалось уравнение. График 5 — Мощность, рассеиваемая во внешнем резисторе, в зависимости от внешних резисторов. График 6 — Мощность, рассеиваемая во внешнем резисторе, в зависимости от внешних резисторов.

Чтобы получить сравнение фактических значений с полученными. Кроме того, мощность, рассеиваемая во внутреннем резисторе, рассчитывалась как функция внешних резисторов. При этом таблица была собрана и, с этими значениями, была построена диаграмма следующим образом.

Экспериментально таблица была создана. Из таблицы был нанесен график. Обратите внимание, что поведение очень близко к поведению кривой графика, показывая удовлетворительные результаты, полученные до сих пор. График 7 — Рассеиваемая во внутреннем резисторе мощность в зависимости от внешних резисторов.

Развиваемая источником полезная мощность выделяется на нагрузке. График зависимости этой мощности от сопротивления нагрузки приведен на рисунке. Эта кривая начинается от пересечения осей координат в нулевой точке, затем возрастает до максимального значения мощности, после чего спадает до нуля при сопротивлении нагрузки равном бесконечности.

Из мощностей, рассеянных во внешних резисторах и внутреннем резисторе, можно определить полную мощность, рассеянную в цепи. Анализируя график, мы имеем, что мощность, рассеиваемая во внешних резисторах, известна как полезная мощность, то есть мощность, исходящая от источника, который фактически будет использоваться нагрузкой, находящейся в цепи. Мощность, рассеиваемая в резисторе, называется рассеиваемой мощностью, так как она заканчивается.

Преобразуется в тепловую энергию внутри самого источника и поэтому бесполезна другим элементам схемы. По-прежнему интерпретируя кривые графа, мы заметили, что кривые, образованные степенями. Таблица 8 — Общая мощность, рассеиваемая в резисторах. Рассеянные во внешних резисторах, имеют максимальную точку. На этом этапе происходит интересное явление, максимальное переключение мощности от источника к внешнему резистору. Чтобы определить эту точку, подставим уравнение в и вычислим его первую производную, приравняв ее к нулю.

Чтобы найти максимальное сопротивление нагрузки, при котором теоретически разовьется максимальная мощность при данном источнике, берется производная от формулы мощности по R и приравнивается к нулю. Максимальная мощность разовьется при сопротивлении внешней цепи, равном внутреннему сопротивлению источника:

Резистивными компонентами схемы, так как большая часть из них преобразуется в тепло в самом источнике, таким образом, «теряется». Когда значения резисторов равны. На равенство между рассеянными силами. При таком равенстве в диссипации предполагается, что суммарная мощность, которая заканчивается, рассеивается на 50% в каждом резисторе. Хотя мы имеем максимальную передачу мощности на внешний резистор, мы понимаем, что эффективность источника не является максимальной. Об этом легко убедиться, увидев равенство диссипации степеней, где половина генерируемой мощности рассеивалась в самом источнике в виде тепла.

Это положение о максимальной мощности при R = r, позволяет экспериментально найти внутреннее сопротивление источника, построив зависимость мощности, выделяемой на нагрузке, от величины сопротивления нагрузки. Найдя реальное, а не теоретическое, сопротивление нагрузки, обеспечивающее максимальную мощность, определяют реальное внутреннее сопротивление источника питания.

КПД источника тока показывает отношение максимальной выделяемой на нагрузке мощности к полной мощности, которую в данный момент развивает

Как найти внутреннее сопротивление формула в физике. Электродвижущая сила. Внутреннее сопротивление источника тока

Цель работы: изучить метод измерения ЭДС и внутреннего сопротивления источника тока с помощью амперметра и вольтметра.

Оборудование: металлический планшет, источник тока, амперметр, вольтметр, резистор, ключ, зажимы, соединительные провода.

Для измерения ЭДС и внутреннего сопротивления источника тока собирают электрическую цепь, схема которой показана на рисунке 1.

К источнику тока подключают амперметр, сопротивление и ключ, соединенные последовательно. Кроме того, непосредствен­но к выходным гнездам источника подключают еще и вольтметр.

ЭДС измеряют по показанию вольтметра при разомкнутом ключе. Этот прием определения ЭДС основан на следствии из за­кона Ома для полной цепи, согласно которому при бесконечно большом сопротивлении внешней цепи напряжение на зажимах источника равно его ЭДС. (См. параграф «Закон Ома для полной цепи» учебника «Физика 10»).

Для определения внутреннего сопротивления источника за­мыкают ключ К. При этом в цепи можно условно выделить два участка: внешний (тот, который подключен к источнику) и внутренний (тот, который находится внутри источника тока). Поскольку ЭДС источника равна сумме падения напряжений на внутрен­нем и внешнем участках цепи:

ε = U r +U R , то U r = ε -U R (1)

По закону Ома для участка цепи U r = I· r (2). Подставив равенство (2) в (1) получают:

I · r = ε U r , откуда r = (ε U R )/ J

Следовательно, чтобы узнать внутреннее сопротивление источника тока, необходимо пред­варительно определить его ЭДС, затем замкнуть ключ и измерить падение напряжения на внеш­нем сопротивлении, а также силу тока в нем.

Ход работы

1. Подготовьте таблицу для записи результатов измерений и вычислений:

ε

U r , B

i,a

r , Ом

    Начертите в тетради схему для измерения ЭДС и внутреннего сопротивления источника.

    После проверки схемы соберите электрическую цепь. Ключ разомкните.

    Измерьте величину ЭДС источника.

    Замкните ключ и определите показания амперметра и вольтметра.

    Вычислите внутреннее сопротивление источника.

  1. Определение эдс и внутреннего сопротивления источника тока графическим методом

Цель работы: изучить измерения ЭДС, внутреннего сопротивления и тока короткого замы­кания источника тока, основанный на анализе графика зависимости напряже­ния на выходе источника от силы тока в цепи.

Оборудование: гальванический элемент, амперметр, вольтметр, резистор R 1 , переменный резистор, ключ, зажимы, металлический планшет, соединительные провода.

Из закона Ома для полной цепи следует, что напряжение на выходе источника тока зависит прямо пропорционально от силы тока в цепи:

так как I =E/(R+r), то IR + Ir = Е, но IR = U, откуда U + Ir = Е или U = Е – Ir (1).

Если построить график зависимости U от I, то по его точкам пересечения с осями координат можно определить Е, I К.З. — силу тока короткого замыкания (ток, который потечет в цепи источни­ка, когда внешнее сопротивление R станет равным нулю).

ЭДС определяют по точке пересечения графика с осью напряжений. Эта точка графика со­ответствует состоянию цепи, при котором ток в ней отсутствует и, следовательно, U = Е.

Силу тока короткого замыкания определяют по точке пересечения графика с осью токов. В этом случае внешнее сопротивление R = 0 и, следовательно, напряжение на выходе источника U = 0.

Внутреннее сопротивление источника находят по тангенсу угла наклона графика относи­тельно оси токов. (Сравните формулу (1) с математической функцией вида У = АХ +В и вспомни­те смысл коэффициента при X).

Ход работы

    Для записи результатов измерений подготовьте таблицу:

  1. После проверки схемы преподавателем соберите электрическую цепь. Ползунок переменного резистора установите в положение, при котором сопротивление цепи, подключенной к источ­нику тока, будет максимальным.
  2. Определите значение силы тока в цепи и напряжение на зажимах источника при максимальной величине сопротивления переменного резистора. Данные измерений занесите в таблицу.

    Повторите несколько раз измерения силы тока и напряжения, уменьшая всякий раз величину переменного сопротивления так, чтобы напряжение на зажимах источника уменьшалось на 0,1В. Измерения прекратите, когда сила тока в цепи достигнет значения в 1А.

    Нанесите полученные в эксперименте точки на график. Напряжение откладывайте по верти­кальной оси, а силу тока — по горизонтальной. Проведите по точкам прямую линию.

    Продолжите график до пересечения с осями координат и определите величины Е и, I К.З.

    Измерьте ЭДС источника, подключив вольтметр к его выводам при разомкнутой внешней це­пи. Сопоставьте значения ЭДС, полученные двумя способами, и укажите причину возможного расхождения результатов.

    Определите внутреннее сопротивление источника тока. Для этого вычислите тангенс угла на­клона построенного графика к оси токов. Так как тангенс угла в прямоугольном треугольнике равен отношению противолежащего катета к прилежащему, то практически это можно сделать, найдя отношение Е / I К.З

На концах проводника, а значит, и тока необходимо наличие сторонних сил неэлектрической природы, с помощью которых происходит разделение электрических зарядов .

Сторонними силами называются любые силы, действующие на электрически заряженные частицы в цепи, за исключением электростатических (т. е. кулоновских).

Сторонние силы приводят в движение заряженные частицы внут-ри всех источников тока: в генераторах, на электростанциях, в гальванических элементах, аккумуляторах и т. д.

При замыкании цепи создается электрическое поле во всех про-водниках цепи. Внутри источника тока заряды движутся под действием сторонних сил против кулоновских сил (электроны движут-ся от положительно заряженного электрода к отрицательному), а во всей остальной цепи их приводит а движение электрическое поле (см. рис. выше).

В источниках тока в процессе работы по разделению заряженных частиц происходит превращение разных видов энергии в электричес-кую. По типу преобразованной энергии различают следующие виды электродвижущей силы:

— электростатическая — в электрофорной машине, в которой происходит превращение механической энергии при трении в электрическую;

— термоэлектрическая — в термоэлементе — внутренняя энергия нагретого спая двух проволок, изготовленных из разных металлов, превращается в электрическую;

— фотоэлектрическая — в фотоэлементе. Здесь происходит превращение энергии света в элек-трическую: при освещении некоторых веществ, например, селена, оксида меди (I) , кремния наблюдается потеря отрицательного электрического заряда;

— химическая — в гальванических элементах, аккумуляторах и др. источниках, в которых происходит превращение химической энергии в электрическую.

Электродвижущая сила (ЭДС) — характеристика источников тока. Понятие ЭДС было введено Г. Омом в 1827 г. для цепей постоянного тока. В 1857 г. Кирхгофф определил ЭДС как работу сторонних сил при переносе единичного электрического заряда вдоль замкнутого контура:

ɛ = A ст /q ,

где ɛ — ЭДС источника тока, А ст — работа сторонних сил , q — количество перемещенного заряда.

Электродвижущую силу выражают в вольтах.

Можно говорить об электродвижущей силе на любом участке цепи. Это удельная работа сторонних сил (работа по перемещению единичного заряда) не во всем контуре, а только на данном участке.

Внутреннее сопротивление источника тока.

Пусть имеется простая замкнутая цепь, состоящая из источника тока (например, гальванического элемента, аккумулятора или генератора) и резистора с сопротивлением R . Ток в замкну-той цепи не прерывается нигде, следовательно, oн существует и внутри источника тока. Любой источник представляет собой некоторое сопротивление дли тока. Оно называется внутренним сопротивлением источника тока и обозначается буквой r .

В генераторе r — это сопротивление обмотки, в гальваническом элементе — сопротивление раствора электролита и электродов.

Таким образом, источник тока характеризуется величинами ЭДС и внутреннего сопротивлении, которые определяют его качество. Например, электростатические машины имеют очень большую ЭДС (до десятков тысяч вольт), но при этом их внутреннее сопротивление огромно (до со-тни Мом). Поэтому они непригодны для получения сильных токов. У гальванических элементов ЭДС всего лишь приблизительно 1 В, но зато и внутреннее сопротивление мало (приблизительно 1 Ом и меньше). Это позволяет с их помощью получать токи, измеряемые амперами.

ЭДС и напряжение. Внутреннее сопротивление источников питания.
Ликбез так ликбез!
Закон Ома. Вот я о чем.
О законе Ома мы уже говорили. Поговорим еще раз — с несколько иной стороны. Не вдаваясь в физические подробности и выражаясь простым кошачьим языком, закон Ома гласит: чем больше э.д.с. (электродвижущая сила), тем больше ток, чем больше сопротивление, тем меньше ток.
Переведя сие заклинание на язык сухих формул получаем:

I=E/R

где:I — сила тока,E — Э.Д.С. — электродвижущая силаR — сопротивление
Ток измеряется в амперах, э.д.с. — в вольтах, а сопротивление носит гордое имя товарища Ома.Э.д.с. — это есть характеристика идеального генератора, внутренне сопротивление которого принято считать бесконечно малым. В реальной жизни такое бывает редко, поэтому в силу вступает закон Ома для последовательной цепи (более знакомый нам):

I=U/R

где:U — напряжение источника непосредственно на его клеммах.
Рассмотрим простой пример.
Представим себе обычную батарейку в виде источника э.д.с. и включенного последовательно с ним некоего резистора, который будет олицетворять собой внутреннее сопротивление батарейки. Подключим параллельно батарейке вольтметр. Его входное сопротивление значительно больше внутреннего сопротивления батарейки, но не бесконечно большое — то есть, через него потечет ток. Величина напряжения, которую покажет вольтметр будет меньше величины э.д.с. как раз на величину падения напряжения на внутреннем воображаемом резисторе при данном токе.Но, тем не менее именно эта величина и принимается за напряжение батарейки.
Формула конечного напряжения при этом будет иметь следующий вид:

U(бат)=E-U(внутр)

Так как со временем у всех элементов питания внутреннее сопротивление увеличивается, то и падение напряжения на внутреннем сопротивлении тоже увеличивается. При этом напряжение на клеммах батарейки уменьшается. Мяу!
Разобрались!
Что же происходит, если вместо вольтметра к батарейке подключить амперметр? Так как собственное сопротивление амперметра стремится к нулю, мы фактически будем измерять ток, протекающий через внутреннее сопротивление батарейки. Так как внутренне сопротивление источника очень небольшое, измеренный при этом ток может достигать н ескольких ампер.
Однако следует заметить, что внутреннее сопротивление источника является таким же элементом цепи, как и все остальные. Поэтому при увеличении тока нагрузки падение напряжения на внутреннем сопротивлении также увеличится, что приводит к уменьшению напряжения на нагрузке. Или как мы, радиокоты, любим выражаться — к просадке напруги.
Чтобы изменение нагрузки как можно меньше влияло на выходное напряжение источника его внутреннее сопротивление стараются свести к минимуму.
Можно так подобрать элементы последовательной цепи, чтобы на каком-нибудь из них получить напряжение, уменьшенное, по сравнению с исходным, во сколько угодно раз.

Допустим, есть простейшая электрическая замкнутая цепь, включающая в себя источник тока, например генератор, гальванический элемент или аккумулятор, и резистор, обладающий сопротивлением R. Поскольку ток в цепи нигде не прерывается, то и внутри источника он течет.

В такой ситуации можно сказать, что любой источник обладает некоторым внутренним сопротивлением, препятствующим току. Это внутреннее сопротивление характеризует источник тока и обозначается буквой r. Для или аккумулятора внутреннее сопротивление — это сопротивление раствора электролита и электродов, для генератора — сопротивление обмоток статора и т. д.

Таким образом, источник тока характеризуется как величиной ЭДС, так и величиной собственного внутреннего сопротивления r – обе эти характеристики свидетельствуют о качестве источника.

Электростатические высоковольтные генераторы (как генератор Ван де Граафа или генератор Уимшурста), к примеру, отличаются огромной ЭДС измеряемой миллионами вольт, при этом их внутреннее сопротивление измеряется сотнями мегаом, потому они и непригодны для получения больших токов.

Гальванические элементы (такие как батарейка) — напротив — имеют ЭДС порядка 1 вольта, хотя внутреннее сопротивление у них порядка долей или максимум — десятка Ом, и от гальванических элементов поэтому можно получать токи в единицы и десятки ампер.

На данной схеме показан реальный источник с присоединенной нагрузкой. Здесь обозначены , его внутреннее сопротивление, а также сопротивление нагрузки. Согласно , ток в данной цепи будет равен:

Поскольку участок внешней цепи однороден, то из закона Ома можно найти напряжение на нагрузке:

Выразив из первого уравнения сопротивление нагрузки, и подставив его значение во второе уравнение, получим зависимость напряжения на нагрузке от тока в замкнутой цепи:

В замкнутом контуре ЭДС равна сумме падений напряжений на элементах внешней цепи и на внутреннем сопротивлении самого источника. Зависимость напряжения на нагрузке от тока нагрузки в идеальном случае линейна.

График это показывает, но экспериментальные данные на реальном резисторе (крестики возле графика) всегда отличаются от идеала:

Эксперименты и логика показывают, что при нулевом токе нагрузки напряжение на внешней цепи равно ЭДС источника, а при нулевом напряжении на нагрузке ток в цепи равен . Это свойство реальных цепей помогает экспериментально находить ЭДС и внутреннее сопротивление реальных источников.

Экспериментальное нахождение внутреннего сопротивления

Чтобы экспериментально определить данные характеристики, строят график зависимости напряжения на нагрузке от величины тока, затем экстраполируют его до пересечения с осями.

В точке пересечения графика с остью напряжения находится значение ЭДС источника, а в точке пересечения с осью тока находится величина тока короткого замыкания. В итоге внутреннее сопротивление находится по формуле:

Развиваемая источником полезная мощность выделяется на нагрузке. График зависимости этой мощности от сопротивления нагрузки приведен на рисунке. Эта кривая начинается от пересечения осей координат в нулевой точке, затем возрастает до максимального значения мощности, после чего спадает до нуля при сопротивлении нагрузки равном бесконечности.

Чтобы найти максимальное сопротивление нагрузки, при котором теоретически разовьется максимальная мощность при данном источнике, берется производная от формулы мощности по R и приравнивается к нулю. Максимальная мощность разовьется при сопротивлении внешней цепи, равном внутреннему сопротивлению источника:

Это положение о максимальной мощности при R = r, позволяет экспериментально найти внутреннее сопротивление источника, построив зависимость мощности, выделяемой на нагрузке, от величины сопротивления нагрузки. Найдя реальное, а не теоретическое, сопротивление нагрузки, обеспечивающее максимальную мощность, определяют реальное внутреннее сопротивление источника питания.

КПД источника тока показывает отношение максимальной выделяемой на нагрузке мощности к полной мощности, которую в данный момент развивает

Попробуем решить эту задачу на конкретном примере. Электродвижущая сила источника питания составляет 4,5 В. К нему подключили нагрузку, и через неё потёк ток, равный 0,26 А. Напряжение при этом стало равным 3,7 В. Первым делом, представим себе, что внутри источника спрятана последовательная цепь из идеального источника напряжения в 4,5 В, внутреннее сопротивление которого равно нулю, а также резистора, номинал которого и требуется найти. Понятно, что на самом деле это не так, но для расчётов аналогия вполне сойдёт.

2 шаг

Запомните, что буквой U обозначают только напряжение под нагрузкой. Для обозначения же электродвижущей силы зарезервирована другая буква – E. Абсолютно точно её измерить невозможно, потому что потребуется вольтметр с бесконечным входным сопротивлением. Даже у электростатического вольтметра (электрометра) оно огромно, но не бесконечно. Но одно дело – абсолютно точно, а другое – с точностью, приемлемой на практике. Второе вполне осуществимо: нужно лишь, чтобы внутреннее сопротивление источника было пренебрежимо мало по сравнению с внутренним сопротивлением вольтметра. А пока суть да дело, посчитаем разницу между ЭДС источника и его напряжением под нагрузкой, потребляющей ток в 260 мА. E-U = 4,5-3,7 = 0,8. Это и будет падение напряжения на том “виртуальном резисторе”.

3 шаг

Ну а дальше всё просто, ибо в дело вступает классический закон Ома. Помним, что ток через нагрузку и “виртуальный резистор” одинаков, ведь они соединены последовательно. Падение напряжения на последнем (0,8 В) делим на силу тока (0,26 А) и получаем 3,08 Ома. Вот и готов ответ! Можно ещё посчитать, какая мощность рассеивается на нагрузке, а какая – бесполезно на источнике. На нагрузке рассеивается: 3,7*0,26=0,962 Вт. На источнике: 0,8*0,26=0,208 Вт. Процентное соотношение между ними вычислите самостоятельно. Но эта не единственный вид задач на нахождение внутреннего сопротивления источника. Есть и такие, в которых вместо силы тока указано сопротивление нагрузки, а остальные исходные данные такие же. Тогда надо вначале проделать ещё одно вычисление. Приведённое в условии напряжение под нагрузкой (не ЭДС!) поделить на сопротивление нагрузки. И получится сила тока в цепи. После чего, как говорят физики, “задача сведена к предыдущей”! Попробуйте составить такую задачу и решить её.

Репетитор-онлайн — подготовка к ЦТ

Пример 11. Шесть одинаковых резисторов по 20 Ом каждый и два конденсатора с электроемкостями 15 и 25 мкФ соединены в цепь так, как показано на рисунке. К концам участка подключают источник с ЭДС, равной 0,23 кВ, и внутренним сопротивлением 3,5 Ом. Найти разность потенциалов между обкладками второго конденсатора.

Решение. Между точками A и Б ток не протекает, так как между этими точками в схему включены конденсаторы. Для определения разности потенциалов между указанными точками упростим схему, исключив из рассмотрения участок АБ.

На рис. а показана схема упрощенной цепи.

Ток течет через резисторы R 1, R 2, R 3, R 4 и R 6, соединенные последовательно. Общее сопротивление такой цепи:

R общ = R 1 + R 2 + R 3 + R 4 + R 6 = 5R,

где R 1 = R 2 = R 3 = R 4 = R 6 = R.

Сила тока I определяется законом Ома для полной цепи:

I=ℰRобщ+r=ℰ5R+r,

где ℰ — ЭДС источника тока, ℰ = 0,23 кВ; r — внутреннее сопротивление источника тока, r = 3,5 Ом; R общ — общее сопротивление цепи, R общ = 5R.

Рассчитаем падение напряжения между точками А и Б.

Между точками А и Б находятся резисторы сопротивлениями R 2, R 3 и R 4, соединенные между собой последовательно, как показано на рис. б.

Их общее сопротивление

R общ1 = R 2 + R 3 + R 4 = 3R.

Падение напряжения на указанных резисторах определяется формулой

U АБ = IR общ1,

или в явном виде, —

UАБ=3ℰR5R+r.

Между точками А и Б включена батарея конденсаторов C 1 и C 2, соединенных между собой последовательно, как показано на рис. в.

Их общая электроемкость

Cобщ=C1C2C1+C2,

где C 1 — электроемкость первого конденсатора, C 1 = 15 мкФ; C 2 — электроемкость второго конденсатора, C 2 = 25 мкФ.

Разность потенциалов на обкладках батареи:

Uобщ=qCобщ,

где q — заряд на обкладках каждого из конденсаторов (совпадает с зарядом батареи при последовательном соединении конденсаторов), q = = C 1U 1 = C 2U 2; U 1 — разность потенциалов между обкладками первого конденсатора; U 2 — разность потенциалов между обкладками второго конденсатора (искомая величина).

В явном виде разность потенциалов между обкладками конденсаторов определяется формулой

Uобщ=C2U2Cобщ=(C1+C2)U2C1.

Падение напряжения на резисторах между точками А и Б совпадает с разностью потенциалов на батарее конденсаторов, подключенной к указанным точкам:

U АБ = U общ.

Данное равенство, записанное в явном виде

3ℰR5R+r=(C1+C2)U2C1,

позволяет получить выражение для искомой величины:

U2=3ℰRC1(5R+r)(C1+C2).

Произведем вычисление:

U2=3⋅0,23⋅103⋅20⋅15⋅10−6(5⋅20+3,5)(15+25)⋅10−6=50 В.

Между обкладками второго конденсатора разность потенциалов составляет 50 В.

Закон Ома

Закон Ома — физический закон, определяющий зависимость между электрическими величинами — напряжением, сопротивлением и током для проводников.
Впервые открыл и описал его в 1826 году немецкий физик Георг Ом, показавший (с помощью гальванометра) количественную связь между электродвижущей силой, электрическим током и свойствами проводника, как пропорциональную зависимость.
Впоследствии свойства проводника, способные противостоять электрическому току на основе этой зависимости, стали называть электрическим сопротивлением (Resistance), обозначать в расчётах и на схемах буквой R и измерять в Омах в честь первооткрывателя.
Сам источник электрической энергии также обладает внутренним сопротивлением, которое принято обозначать буквой r.

Закон Ома для участка цепи

Со школьного курса физики всем хорошо известна классическая трактовка Закона Ома:

Сила тока в проводнике прямо пропорциональна напряжению на концах проводника и обратно пропорциональна его сопротивлению.

I = U/R

Это значит, если к концам проводника сопротивлением R = 1 Ом приложено напряжение U = 1 Вольт, тогда величина тока I в проводнике будет равна 1/1 = 1 Ампер.

Отсюда следуют ещё два полезных соотношения:

Если в проводнике, сопротивлением 1 Ом, протекает ток 1 Ампер, значит на концах проводника напряжение 1 Вольт (падение напряжения).

U = IR

Если на концах проводника есть напряжение 1 Вольт и по нему протекает ток 1 Ампер, значит сопротивление проводника равно 1 Ом.

R = U/I

Вышеописанные формулы в таком виде могут быть применимы для переменного тока лишь в том случае, если цепь состоит только из активного сопротивления R.
Кроме того, следует помнить, что Закон Ома справедлив только для линейных элементов цепи.

Предлагается простой Онлайн-калькулятор для практических расчётов.

Закон Ома. Расчёт напряжения, сопротивления, тока, мощности.
После сброса ввести два любых известных параметра.

I=U/R;   U=IR;   R=U/I;
P=UI   P=U²/R;   P=I²R;
R=U²/P;   R=P/I²   U=√(PR)   I= √(P/R)




Закон Ома для замкнутой цепи

Если к источнику питания подключить внешнюю цепь сопротивлением R, в цепи пойдёт ток с учётом внутреннего сопротивления источника:

I — Сила тока в цепи.
— Электродвижущая сила (ЭДС) — величина напряжения источника питания не зависящая от внешней цепи (без нагрузки). Характеризуется потенциальной энергией источника.
r — Внутреннее сопротивление источника питания.

Для электродвижущей силы внешнеее сопротивление R и внутреннее r соединены последовательно, значит величина тока в цепи определится значением ЭДС и суммой сопротивлений: I = /(R+r) .

Напряжение на выводах внешней цепи определится исходя из силы тока и сопротивления R соотношением, которое уже рассматривалось выше: U = IR.
Напряжение U, при подключении нагрузки R, всегда будет меньше чем ЭДС на величину произведения I*r, которую называют падением напряжения на внутреннем сопротивлении источника питания.
С этим явлением мы сталкиваемся достаточно часто, когда видим в работе частично разряженные батарейки или аккумуляторы.
По мере разряда, увеличивается их внутреннее сопротивление, следовательно, увеличивается падение напряжение внутри источника, значит уменьшается внешнее напряжение U = — I*r.
Чем меньше ток и внутреннее сопротивление источника, тем ближе по значению его ЭДС и напряжение на его выводах U.
Если ток в цепи равен нулю, следовательно, = U. Цепь разомкнута, ЭДС источника равна напряжению на его выводах.

В случаях, когда внутренним сопротивлением источника можно пренебречь (r ≈ 0), напряжение на выводах источника будет равно ЭДС ( ≈ U ) независимо от сопротивления внешней цепи R.
Такой источник питания называют источником напряжения.


Закон Ома для переменного тока

При наличии индуктивности или ёмкости в цепи переменного тока необходимо учитывать их реактивное сопротивление.
В таком случае запись Закона Ома будет иметь вид:

I = U/Z

Здесь Z — полное (комплексное) сопротивление цепи — импеданс. В него входит активная R и реактивная X составляющие.
Реактивное сопротивление зависит от номиналов реактивных элементов, от частоты и формы тока в цепи.
Более подробно ознакомится с комплексным сопротивлением можно на страничке импеданс.

С учётом сдвига фаз φ, созданного реактивными элементами, для синусоидального переменного тока обычно записывают Закон Ома в комплексной форме:

— комплексная амплитуда тока. = Iampe
— комплексная амплитуда напряжения. = Uampe
— комплексное сопротивление. Импеданс.
φ — угол сдвига фаз между током и напряжением.
e — константа, основание натурального логарифма.
j — мнимая единица.
Iamp , Uamp — амплитудные значения синусоидального тока и напряжения.

Нелинейные элементы и цепи

Закон Ома не является фундаментальным законом природы и может быть применим в ограниченных случаях, например, для большинства проводников.
Его невозможно использовать для расчёта напряжения и тока в полупроводниковых или электровакуумных приборах, где эта зависимость не является пропорциональной и её можно определять только с помощью вольтамперной характеристики (ВАХ). К данной категории элементов относятся все полупроводниковые приборы (диоды, транзисторы, стабилитроны, тиристоры, варикапы и т.д.) и электронные лампы.
Такие элементы и цепи, в которых они используются, называют нелинейными.

Похожие статьи: Постоянный ток. Переменный ток.


Замечания и предложения принимаются и приветствуются!

Формула определения эдс. Формула эдс индукции. Внутреннее сопротивление источника тока

На концах проводника, а значит, и тока необходимо наличие сторонних сил неэлектрической природы, с помощью которых происходит разделение электрических зарядов .

Сторонними силами называются любые силы, действующие на электрически заряженные частицы в цепи, за исключением электростатических (т. е. кулоновских).

Сторонние силы приводят в движение заряженные частицы внут-ри всех источников тока: в генераторах, на электростанциях, в гальванических элементах, аккумуляторах и т. д.

При замыкании цепи создается электрическое поле во всех про-водниках цепи. Внутри источника тока заряды движутся под действием сторонних сил против кулоновских сил (электроны движут-ся от положительно заряженного электрода к отрицательному), а во всей остальной цепи их приводит а движение электрическое поле (см. рис. выше).

В источниках тока в процессе работы по разделению заряженных частиц происходит превращение разных видов энергии в электричес-кую. По типу преобразованной энергии различают следующие виды электродвижущей силы:

— электростатическая — в электрофорной машине, в которой происходит превращение механической энергии при трении в электрическую;

— термоэлектрическая — в термоэлементе — внутренняя энергия нагретого спая двух проволок, изготовленных из разных металлов, превращается в электрическую;

— фотоэлектрическая — в фотоэлементе. Здесь происходит превращение энергии света в элек-трическую: при освещении некоторых веществ, например, селена, оксида меди (I) , кремния наблюдается потеря отрицательного электрического заряда;

— химическая — в гальванических элементах, аккумуляторах и др. источниках, в которых происходит превращение химической энергии в электрическую.

Электродвижущая сила (ЭДС) — характеристика источников тока. Понятие ЭДС было введено Г. Омом в 1827 г. для цепей постоянного тока. В 1857 г. Кирхгофф определил ЭДС как работу сторонних сил при переносе единичного электрического заряда вдоль замкнутого контура:

ɛ = A ст /q ,

где ɛ — ЭДС источника тока, А ст — работа сторонних сил , q — количество перемещенного заряда.

Электродвижущую силу выражают в вольтах.

Можно говорить об электродвижущей силе на любом участке цепи. Это удельная работа сторонних сил (работа по перемещению единичного заряда) не во всем контуре, а только на данном участке.

Внутреннее сопротивление источника тока.

Пусть имеется простая замкнутая цепь, состоящая из источника тока (например, гальванического элемента, аккумулятора или генератора) и резистора с сопротивлением R . Ток в замкну-той цепи не прерывается нигде, следовательно, oн существует и внутри источника тока. Любой источник представляет собой некоторое сопротивление дли тока. Оно называется внутренним сопротивлением источника тока и обозначается буквой r .

В генераторе r — это сопротивление обмотки, в гальваническом элементе — сопротивление раствора электролита и электродов.

Таким образом, источник тока характеризуется величинами ЭДС и внутреннего сопротивлении, которые определяют его качество. Например, электростатические машины имеют очень большую ЭДС (до десятков тысяч вольт), но при этом их внутреннее сопротивление огромно (до со-тни Мом). Поэтому они непригодны для получения сильных токов. У гальванических элементов ЭДС всего лишь приблизительно 1 В, но зато и внутреннее сопротивление мало (приблизительно 1 Ом и меньше). Это позволяет с их помощью получать токи, измеряемые амперами.

«Физика — 10 класс»

Любой источник тока характеризуется электродвижущей силой, или сокращённо ЭДС. Так, на круглой батарейке для карманного фонарика написано: 1,5 В.
Что это значит?

Если соединить проводником два разноимённо заряженных шарика, то заряды быстро нейтрализуют друг друга, потенциалы шариков станут одинаковыми, и электрическое поле исчезнет (рис. 15.9, а).

Сторонние силы.

Для того чтобы ток был постоянным, надо поддерживать постоянное напряжение между шариками. Для этого необходимо устройство (источник тока), которое перемещало бы заряды от одного шарика к другому в направлении, противоположном направлению сил, действующих на эти заряды со стороны электрического поля шариков. В таком устройстве на заряды, кроме электрических сил, должны действовать силы неэлектростатического происхождения (рис. 15.9, б). Одно лишь электрическое поле заряженных частиц (кулоновское поле ) не способно поддерживать постоянный ток в цепи.

Любые силы, действующие на электрически заряженные частицы, за исключением сил электростатического происхождения (т. е. кулоновских), называют сторонними силами .

Вывод о необходимости сторонних сил для поддержания постоянного тока в цепи станет ещё очевиднее, если обратиться к закону сохранения энергии.

Электростатическое поле потенциально. Работа этого поля при перемещении в нём заряженных частиц по замкнутой электрической цепи равна нулю. Прохождение же тока по проводникам сопровождается выделением энергии — проводник нагревается. Следовательно, в цепи должен быть какой-то источник энергии, поставляющий её в цепь. В нём, помимо кулоновских сил, обязательно должны действовать сторонние, непотенциальные силы. Работа этих сил вдоль замкнутого контура должна быть отлична от нуля.

Именно в процессе совершения работы этими силами заряженные частицы приобретают внутри источника тока энергию и отдают её затем проводникам электрической цепи.

Сторонние силы приводят в движение заряженные частицы внутри всех источников тока: в генераторах на электростанциях, в гальванических элементах, аккумуляторах и т. д.

При замыкании цепи создаётся электрическое поле во всех проводниках цепи. Внутри источника тока заряды движутся под действием сторонних сил против кулоновских сил (электроны от положительно заряженного электрода к отрицательному), а во внешней цепи их приводит в движение электрическое поле (см. рис. 15.9, б).

Природа сторонних сил.

Природа сторонних сил может быть разнообразной. В генераторах электростанций сторонние силы — это силы, действующие со стороны магнитного поля на электроны в движущемся проводнике.

В гальваническом элементе, например в элементе Вольта, действуют химические силы.

Элемент Вольта состоит из цинкового и медного электродов, помещённых в раствор серной кислоты. Химические силы вызывают растворение цинка в кислоте. В раствор переходят положительно заряженные ионы цинка, а сам цинковый электрод при этом заряжается отрицательно. (Медь очень мало растворяется в серной кислоте.) Между цинковым и медным электродами появляется разность потенциалов, которая и обусловливает ток во внешней электрической цепи.

Действие сторонних сил характеризуется важной физической величиной, называемой электродвижущей силой (сокращённо ЭДС).

Электродвижущая сила источника тока равна отношению работы сторонних сил при перемещении заряда по замкнутому контуру к абсолютной величине этого заряда:

Электродвижущую силу как и напряжение, выражают в вольтах.

Разность потенциалов на клеммах батареи при разомкнутой цепи равна электродвижущей силе. ЭДС одного элемента батареи обычно 1-2 В.

Можно говорить также об электродвижущей силе и на любом участке цепи. Это удельная работа сторонних сил (работа по перемещению единичного заряда) не во всём контуре, а только на данном участке.

Электродвижущая сила гальванического элемента есть величина, численно равная работе сторонних сил при перемещении единичного положительного заряда внутри элемента от одного полюса к другому.

Работа сторонних сил не может быть выражена через разность потенциалов, так как сторонние силы непотенциальны и их работа зависит от формы траектории перемещения зарядов.

ЭДС (ε ) — отношение работы сторонних сил по разделению зарядов к величине этого заряда, иначе, способность данного источника давать необходимое количество зарядов необходимой энергии.

— ЭДС.
ЭДС не является силой в Ньютоновом смысле (неудачное название величины, сохраненное как дань традиции).
ε i возникает при изменении магнитного потока Ф , пронизывающего контур.

Дополнительно см. презентацию «Электромагнитная индукция» , а также видеофильмы «Электромагнитная индукция «, «Опыт Фарадея «, мультфильмы «Электромагнитная индукция «, «Вращение рамки в магнитном поле (генератор) «

— ЭДС индукции.

— ЭДС индукции при движении одного из проводников контура (так, чтобы менялся Ф). В этом случае проводник длиной l , движущийся со скоростью v становится источником тока.

— ЭДС индукции в контуре, вращающемся в магнитном поле со скоростью ω.

Другие формулы, где встречается ЭДС:

— закон Ома для полной цепи. В замкнутой цепи ЭДС рождает электрический ток I.

Направление индукционного тока определяют по правилам:
— правило Ленца — возникающий в замкнутом контуре индукционный ток противо действует тому изменению магнитного потока, которым вызван данный ток;
— для проводника, движущегося в магнитном поле, иногда проще воспользоваться правилом правой руки — если расположить раскрытую ладонь правой руки так, чтобу в нее входили силовые линии магнитного поля В , а большой палец , отставленный в сторону указывал направление скорости v , то четыре пальца руки укажут направление индукционного тока I .

— ЭДС самоиндукции при изменении тока в проводнике.

Электродвижущая сила, в народе ЭДС, также как и напряжение измеряется в вольтах, но носит совсем иной характер.

ЭДС с точки зрения гидравлики

Думаю, вам уже знакома водонапорная башня из прошлой статьи про

Допустим, что башня полностью заполнена водой. Снизу башни мы просверлили отверстие и врезали туда трубу, по которой вода бежит к вам домой.


Сосед захотел полить огурцы, вы решили помыть автомобиль, мать затеяла стирку и вуаля! Поток воды стал меньше и меньше, и вскоре совсем иссяк… Что случилось? Закончилась вода в башне…


Время, которое потребуется, чтобы опустошить башню, зависит от емкости самой башни, а также от того, сколько потребителей будут пользоваться водой.

Все то же самое можно сказать и про радиоэлемент конденсатор :

Допустим мы его зарядили от батарейки 1,5 вольта и он принял заряд. Нарисуем заряженный конденсатор вот так:

Но как только мы цепляем к нему нагрузку (пусть нагрузкой будет светодиод) с помощью замыкания ключа S, в первые доли секунд светодиод будет светиться ярко, а потом тихонько угасать… и пока полностью не потухнет. Время угасания светодиода будет зависеть от емкости конденсатора, а также от того, какую нагрузку мы цепляем к заряженному конденсатору.

Как я уже сказал, это равносильно простой наполненной башне и потребителям, которые пользуются водой.

Но почему тогда в наших башнях вода никогда не заканчивается? Да потому что работает насос подачи воды ! А откуда этот насос берет воду? Из скважины, которая пробурена для добычи подземных вод. Иногда ее еще называют артезианской.


Как только башня полностью наполнится водой, насос выключается. В наших водобашнях насос всегда поддерживает максимальный уровень воды.

Итак, давайте вспомним, что такое напряжение ? По аналогии с гидравликой – это уровень воды в водобашне. Полная башня – это максимальный уровень воды, значит максимальное напряжение. Нет в башне воды – напряжение ноль.

ЭДС электрического тока

Как вы помните из прошлых статей, молекулы воды – это “электроны”. Для возникновения электрического тока, электроны должны двигаться в одном направлении. Но чтобы они двигались в одном направлении, должно быть напряжение и какая-нибудь нагрузка. То есть вода в башне – это напряжение, а люди, которые тратят воду для своих нужд – это нагрузка, так как они создают поток воды из трубы, которая находится у подножия водобашни. А поток – это не что иное, как сила тока.

Также должно соблюдаться условие, что вода должна всегда быть на максимальной отметке, независимо от того, сколько людей тратит ее для своих нужд одновременно, иначе башня опустошится. Для водобашни этим спасительным средством является водонасос. А для электрического тока?

Для электрического тока должна быть какая-то сила, которая бы толкала электроны в одном направлении в течение продолжительного времени. То есть эта сила должна двигать электроны! Электродвижущая сила! Да, именно так! ЭЛЕКТРОДВИЖУЩАЯ СИЛА! Можно назвать ее сокращенно ЭДС – Э лектро Д вижущая С ила. Измеряется она в вольтах, как и напряжение, и обозначается в основном буквой E .

Значит, в наших батарейках тоже есть такой “насос”? Есть, и правильней было бы его назвать “насос подачи электронов”). Но, конечно, так никто не говорит. Говорят просто – ЭДС. Интересно, а где спрятан этот насос в батарейке? Это просто-напросто электрохимическая реакция, из-за которой держится “уровень воды” в батарейке, но потом все-таки этот насос изнашивается и напряжение в батарейке начинает проседать, потому как “насос” не успевает качать воду. В конце концов он полностью ломается и напряжение на батарейке стает практически ноль.

Реальный источник ЭДС

Источник электрической энергии – это источник ЭДС с внутренним сопротивлением R вн. Это могут быть какие-либо химические элементы питания, наподобие батареек и аккумуляторов


Их внутреннее строение с точки зрения ЭДС выглядит примерно вот так:


Где E – это ЭДС, а R вн – это внутреннее сопротивление батарейки

Итак, какие выводы можно сделать из этого?

Если к батарейке не цепляется никакая нагрузка, типа лампы накаливания и тд, то в результате сила тока в такой цепи будет равняться нулю. Упрощенная схема будет такой:


Но если мы все-таки присоединим к нашей батарейке лампочку накаливания, то у нас цепь станет замкнутой и в цепи будет течь ток:

Если начертить график зависимости силы в цепи тока от напряжения на батарейке, то он будет выглядеть вот так:


Какой напрашивается вывод? Для того, чтобы замерить ЭДС батарейки, нам достаточно просто взять хороший мультиметр с высоким входным сопротивлением и замерять напряжение на клеммах батарейки.

Идеальный источник ЭДС

Допустим, пусть наша батарейка обладает нулевым внутренним сопротивлением, тогда получается, что R вн =0.

Нетрудно догадаться, что в этом случае падение напряжение на нулевом сопротивлении также будет равняться нулю. В результате, наш график примет вот такой вид:


В результате мы получили просто источник ЭДС. Следовательно, источник ЭДС – это идеальный источник питания, у которого напряжение на клеммах не зависит от силы тока в цепи. То есть, какую нагрузку мы бы не цепляли на такой источник ЭДС, у нас он все равно будет выдавать положенное напряжение без просадки. Сам источник ЭДС обозначается вот так:

На практике идеального источника ЭДС не существует.

Типы ЭДС

электрохимическая (ЭДС батареек и аккумуляторов)

фотоэффекта (получение электрического тока от солнечной энергии)

индукции (генераторы, использующие принцип электромагнитной индукции)

Эффект Зеебека или термоЭДС (возникновение электрического тока в замкнутой цепи, состоящей из последовательно соединённых разнородных проводников , контакты между которыми находятся при различных температурах)

пьезоЭДС (получение ЭДС от )

Для поддержания электрического тока в проводнике требуется внешний источник энергии, создающий все время разность потенциалов между концами этого проводника. Такие источники энергии получили название источников электрической энергии (или источников тока).

Источники электрической энергии обладают определенной электродвижущей силой (сокращенно ЭДС ), которая создает и длительное время поддерживает разность потенциалов между концами проводника. Иногда говорят, что ЭДС создает электрический ток в цепи. Нужно помнить об условности такого определения, так как выше мы уже установили, что причина возникновения и существования электрического тока — электрическое поле.

Источник электрической энергии производит определенную работу, перемещая электрические заряды по всей замкнутой цепи..

Определение: Работа, совершаемая источником электрической энергии при переносе единицы положительного заряда по всей замкнутой цепи, называется ЭДС источника

За единицу измерения электродвижущей силы принят вольт (сокращенно вольт обозначается буквой В или V — «вэ» латинское).

ЭДС источника электрической энергии равна одному вольту, если при перемещении одного кулона электричества по всей замкнутой, цепи источник электрической энергии совершает работу, равную одному джоулю:

В практике для измерения ЭДС используются как более крупные, так и более мелкие единицы, а именно:

1 киловольт (кВ, kV), равный 1000 В;

1 милливольт (мВ, mV), равный одной тысячной доле вольта (10-3 В),

1 микровольт (мкВ, μV), равный одной миллионной доле вольта (10-6 В).

Очевидно, что 1 кВ = 1000 В; 1 В = 1000 мВ = 1 000 000 мкВ; 1 мВ= 1000 мкВ.

В настоящее, время существует несколько видов источников электрической энергии. Впервые в качестве источника электрической энергии была использована гальваническая батарея, состоящая из нескольких цинковых и медных кружков, между которыми была проложена кожа, смоченная в подкисленной воде. В гальванической батарее химическая энергия превращалась в электрическую (подробнее об этом будет рассказано в главе XVI). Свое название гальваническая батарея получила по имени итальянского физиолога Луиджи Гальвани (1737-1798), одного из основателей учения об электричестве.

Многочисленные опыты по усовершенствованию и практическому использованию гальванических батарей были проведены русским ученым Василием Владимировичем Петровым. Еще в начале прошлого века он создал самую большую в мире гальваническую батарею и использовал ее для ряда блестящих опытов.

Источники электрической энергии, работающие по принципу преобразования химической энергии в электрическую, называются химическими источниками электрической энергии.

Другим основным источником электрической энергий, получившим широкое применение в электротехнике и радиотехнике, является генератор. В генераторах механическая энергия преобразуется в электрическую.

На электрических схемах источники электрической энергии и генераторы обозначаются так, как это показано на рис. 1.

Рисунок 1. Условные обозначения источников электрической энергии: а — источник ЭДС, общее обозначение, б — источник тока, общее обозначение; в — химический источник электрической энергии; г — батарея химических источников; д — источник потоянного напряжения; е — источник переменного нарияжения; ж — генератор.

У химических источников электрической энергии и у генераторов электродвижущая сила проявляется одинаково, создавая на зажимах источника разность потенциалов и поддерживая ее длительное время. Эти зажимы называются полюсами источника электрической энергии . Один полюс источника электрической энергии имеет положительный потенциал (недостаток электронов), обозначается знаком плюс (+) и называется положительным полюсом. Другой полюс имеет отрицательный потенциал (избыток электронов), обозначается знаком минус (-) и называется отрицательным полюсом.

От источников электрической энергии электрическая энергия передается по проводам к ее потребителям (электрические лампы, электродвигатели, электрические дуги, электронагревательные приборы и т. д.).

Определение : Совокупность источника электрической энергии, ее потребителя и соединительных проводов называется электрической цепью.

Простейшая электрическая цепь показана на рис. 2.

Рисунок 2. Б — источник электрической энергии; SA — выключатель; EL — потребитель электрической энергии (лампа).

Для того чтобы по цепи проходил электрический ток, она должна быть замкнутой. По замкнутой электрической цепи непрерывно проходит ток, так как между полюсами источника электрической энергии существует некоторая разность потенциалов. Эта разность потенциалов называется напряжением источника и обозначается буквой U . Единицей измерения напряжения служит вольт. Так же как и ЭДС, напряжение может измеряться в киловольтах, милливольтах и микровольтах.

Для измерения величины ЭДС и напряжения применяется прибор, называемый вольтметром . Если вольтметр подключить непосредственно к полюсам источника электрической энергии, то при разомкнутой электрической цепи он покажет ЭДС источника электрической энергии, а при замкнутой — напряжение на его зажимах: (рис. 3).

Рисунок 3. Измерение ЭДС и напряжения источника электрической энергии: а- измерение ЭДС источника электрической энергии; б — измерение напряжения на зажимах источника электрической энергии..

Заметим, что напряжение на зажимах источника электрической энергии всегда меньше его ЭДС.

Нахождение внутреннего сопротивления и ЭДС источника.

В статье расчёт в маткаде переходных процессов в ёмкостном фильтре исследовался переходный процесс в фильтре поставленном на выходе однофазного однополупериодного выпрямителя, при этом в схеме замещения выпрямитель с источником переменного напряжения заменены последовательным соединением источника ЭДС и резистора, такая замена делает возможным расчёт схем но при этом для расчётов требуется найти ЭДС источника и его внутреннее сопротивление. Найти ЭДС источника и его внутреннее сопротивление эксперементально можно проделав опыт холостого хода и опыт короткого замыкания но это не всегда возможно, например когда необходимо найти ЭДС и внутреннее сопротивление источника представляющего собой вторичную обмотку трансформатора, поэтому бывает необходимо определить параметры схемы замещения источника не внося больших изменений сопротивления нагрузки в схему. Рассмотрим схему на рисунке 1:

Рисунок 1 — Схема для определения параметров схемы замещения источника.

В этой схеме значения ЭДС источника и его внутреннего сопротивления неизвестны, известны только показания амперметра и вольтметра. Учтём что тока в цепи вольтметра нет, так как у него большое сопротивление и его проводимостью можно пренебреч а сопротивление амперметра настолько мало что им тоже можно пренебреч и заменить амперметр перемычкой. Ток в этой цепи обозначим как I1 (его показывает амперметр) а напряжение на R1 и G обозначим как U1 (его показывает вольтметр) при этом будем считать что ток направлен как показано на рисунке 1, а напряжения на R1 и r направлены в туже сторону что и ток. Рассмотрим схему на рисунке 2 в которой изменено (в нашем случае увеличено) сопротивление реостата:

Рисунок 2 — Схема для определения параметров схемы замещения источника с изменённым сопротивлением реостата.

В этой схеме показание амперметра обозначим как I2 а показание вольтметра как U2.

Из схемы на рисунке 1, составим уравнение по второму закону Кирхгофа для контура который остаётся если заменить вольтметр разрывом:

Здесь E — ЭДС источника, U1 — напряжение на реостате (показывает вольтметр), I1 — ток в цепи (показывает амперметр), r — внутреннее сопротивление источника. Выразим из уравнения (1) напряжение U1:

Аналогично найдём U2, используя схему на рисунке 2:

Подставим (1) в (3):

Выразим из уравнения (4) внутреннее сопротивление источника r:

 Подставим (6) в (1) и найдём ЭДС источника:

По формулам (6) и (7) находятся параметры схемы замещения источника электрической энергии (по формуле (7) его ЭДС, по формуле (6) его внутреннее сопротивление). Последовательно с реостатом можно поставить измерительный резистор и использовать его для измерения тока вольтметром тогда измерения можно проводить одним вольтметром сначала подключая его паралельно источнику G, а потом паралельно измерительному резистору.
Для расчёта внутреннего сопротивления и ЭДС источника можно воспользоваться программой:
Первое измерение должно быть с меньшим сопротивлением реостата, а второе с большим.

Закон ома для внутренней цепи

Закон Ома для полной цепи – эмпирический (полученный из эксперимента) закон, который устанавливает связь между силой тока, электродвижущей силой (ЭДС) и внешним и внутренним сопротивлением в цепи.

При проведении реальных исследований электрических характеристик цепей с постоянным током необходимо учитывать сопротивление самого источника тока. Таким образом в физике осуществляется переход от идеального источника тока к реальному источнику тока, у которого есть свое сопротивление (см. рис. 1).

Рис. 1. Изображение идеального и реального источников тока

Рассмотрение источника тока с собственным сопротивлением обязывает использовать закон Ома для полной цепи.

Сформулируем закона Ома для полной цепи так (см. рис. 2): сила тока в полной цепи прямо пропорциональна ЭДС и обратно пропорциональна полному сопротивлению цепи, где под полным сопротивлением понимается сумма внешних и внутренних сопротивлений.

Рис. 2. Схема закона Ома для полной цепи.

Формула закона Ома для полной цепи

Рассмотрим некоторые задачи на данную тему. Задачи на закон Ома для полной цепи, как правило, дают ученикам 10 класса, чтобы они могли лучше усвоить указанную тему.

I. Определите силу тока в цепи с лампочкой, сопротивлением 2,4 Ом и источником тока, ЭДС которого равно 10 В, а внутреннее сопротивление 0,1 Ом.

По определению закона Ома для полной цепи, сила тока равна:

II. Определить внутреннее сопротивление источника тока с ЭДС 52 В. Если известно, что при подключении этого источника тока к цепи с сопротивлением 10 Ом амперметр показывает значение 5 А.

Запишем закон Ома для полной цепи и выразим из него внутреннее сопротивление:

III. Однажды школьник спросил у учителя по физике: «Почему батарейка садится?» Как грамотно ответить на данный вопрос?

Мы уже знаем, что реальный источник обладает собственным сопротивлением, которое обусловлено либо сопротивлением растворов электролитов для гальванических элементов и аккумуляторов, либо сопротивлением проводников для генераторов. Согласно закону Ома для полной цепи:

следовательно, ток в цепи может уменьшаться либо из-за уменьшения ЭДС, либо из-за повышения внутреннего сопротивления. Значение ЭДС у аккумулятора почти постоянный. Следовательно, ток в цепи понижается за счет повышения внутреннего сопротивления. Итак, «батарейка» садится, так как её внутреннее сопротивление увеличивается.

Немецкий физик Георг Симон Ом (1787—1854) открыл основной закон электрической цепи.

Закон Ома для участка цепи:

Определение: Cила тока I на участке электрической цепи прямо пропорциональна напряжению U на концах участка и обратно пропорциональна его сопротивлению R.

  1. I — сила тока (в системе СИ измеряется — Ампер)
  2. Сила тока в проводнике прямо пропорциональна напряжению и обратно пропорциональна сопротивлению.
  3. Формула: I=frac
  4. U — напряжение (в системе СИ измеряется — Вольт)

Падение напряжения на участке проводника равно произведению силы тока в проводнике на сопротивление этого участка.

Формула: U=IR

  • R— электрическое сопротивление (в системе СИ измеряется — Ом).
  • Электрическое сопротивление R это отношение напряжения на концах проводника к силе тока, текущего по проводнику.
  • Формула R=frac
  • Определение единицы сопротивления — Ом

    1 Ом представляет собой электрическое сопротивление участка проводника, по которому при напряжении 1(Вольт) протекает ток 1 (Ампер).

    Закон Ома для полной цепи

    Определение: Сила тока в цепи пропорциональна действующей в цепи ЭДС и обратно пропорциональна сумме сопротивлений цепи и внутреннего сопротивления источника

    Формула I=frac <varepsilon>

    • varepsilon — ЭДС источника напряжения, В;
    • I — сила тока в цепи, А;
    • R — сопротивление всех внешних элементов цепи, Ом;
    • r — внутреннее сопротивление источника напряжения, Ом.

    Как запомнить формулы закона Ома

    Треугольник Ома поможет запомнить закон. Нужно закрыть искомую величину, и два других символа дадут формулу для её вычисления.

    .

    • U — электрическое напряжение;
    • I — сила тока;
    • P — электрическая мощность;
    • R — электрическое сопротивление

    Смотри также:

    Для закрепления своих знаний решай задания и варианты ЕГЭ по физике с ответами и пояснениями.

    Всякая простейшая электрическая цепь состоит из источника электрической энергии (гальванического элемента, аккумулятора, генератора и т. п.), потребителей или приемников электрической энергии (ламп накаливания, электронагревательных приборов, электродвигателей и т. п.) и соединительных проводов, соединяющих источник электрической энергии с потребителем.

    Источник электрической энергии дает электрическую энергию, а потребитель эту энергию преобразует в другие виды энергии: свет, тепло, движение и т. д.

    Источник тока имеет сопротивление r, называемое внутренним сопротивлением цепи.

    Резистор имеет сопротивление R, называемое внешним сопротивлением цепи.

    Любая полная электрическая цепь состоит из двух частей – внешней и внутренней.

    Внутренняя часть цепи состоит из источника.

    Всё остальные элементы цепи являются внешними.

    Полную цепь можно рассматривать как последовательное соединение сопротивления внешней цепи и внутреннего сопротивления источника тока.


    Закон Ома для участка цепи

    Сила тока на участке цепи, не включающем источник тока, прямо пропорциональна приложенному напряжению и обратно пропорциональна сопротивлению проводника.

    Закон Ома справедлив для металлических проводников, угля и электролитов.

    Закон Ома несправедлив для газов (если только не ограничиться небольшими напряжениями, которые ещё не сказываются на сопротивлении ионизированного газа), так как сопротивление газов, сделанных проводящими (например, ионизированных нагревом), зависит от приложенного напряжения.

    Закон Ома для замкнутой цепи

    Сила тока в замкнутой цепи прямо пропорциональна ЭДС источника и обратно пропорциональна полному сопротивлению цепи.

    I – сила тока; ε – электродвижущая сила; R – внешнее сопротивление цепи; r – внутреннее сопротивление цепи.

    Не нашли то, что искали? Воспользуйтесь поиском:

    Лучшие изречения: Только сон приблежает студента к концу лекции. А чужой храп его отдаляет. 8852 — | 7556 — или читать все.

    91.146.8.87 © studopedia.ru Не является автором материалов, которые размещены. Но предоставляет возможность бесплатного использования. Есть нарушение авторского права? Напишите нам | Обратная связь.

    Отключите adBlock!
    и обновите страницу (F5)

    очень нужно

    Формула внутреннего сопротивления

    — объяснение, примеры с решениями что термин, обозначаемый буквой e = ЭДС, известный как электродвижущая сила вольт, записывается как: I = ток, который обозначается буквой A, буква R = сопротивление нагрузки, а буква r — внутреннее сопротивление измеренного элемента. в ом.

    При преобразовании приведенного выше уравнения мы получаем следующее:

    То есть e = IR + Ir или, e = V + Ir

    Формула внутреннего сопротивления

    В приведенном выше уравнении мы можем сказать, что буква V представляет собой клемма разности потенциалов на ячейке, когда через цепь протекает ток, обозначенный буквой I.

    Мы можем отметить: ЭДС, обозначенная буквой e ячейки, всегда больше, чем разность потенциалов, обычно терминальная по всей ячейке.

    Пример: 1 — это разность потенциалов, которая проходит через ячейку, когда по цепи не течет ток, равный 3 В. Когда течет ток, выделяемый I = 0,37, то есть в амперах, это конечная разность потенциалов, которая падает до 2,8 Вольт. Определить внутреннее сопротивление, обозначенное буквой r ячейки?

    Решение:

    То есть e = V + Ir

    Или мы видим e — V = Ir

    Или это (e — V) / I = r

    Следовательно, мы видим, что r = (3.0 — 2,8) / 0,37 = 0,54 Ом.

    Теперь, благодаря внутреннему сопротивлению ячейки, электроны, движущиеся через ячейку, превращают часть электрической энергии в тепловую. Таким образом, мы видим, что разность потенциалов доступна для остальной части схемы, а именно:

    То есть V = E, что означает ЭДС ячейки — Ir, что является п.о. через внутренний резистор

    Электродвижущая сила, обозначенная буквой e или э.д.с. это энергия, которая обычно вырабатывается элементом или батареей на кулон заряда, проходящего через них.Таким образом, мы можем сказать, что он измеряется в вольтах, то есть V. Считается, что он равен разности потенциалов, которая возникает на выводах ячейки, когда ток не течет.

    ε = E / Q

    • Можно сказать, что e = электродвижущая сила в вольтах, В

    • И затем E = энергия в джоулях, Дж

    • Тогда буква Q = заряд в кулонах, C

    Батареи и элементы имеют внутреннее сопротивление, обозначенное буквой r, которое измеряется в омах и обозначается W.Когда электрический ток проходит по цепи, внутреннее сопротивление, которое имеет сама ячейка, сопротивляется потоку тока и настолько тепловое, что считается теплом, т. Е. Энергия теряется в самой ячейке.

    ε = I (R + r)

    • Буква e = электродвижущая сила в вольтах, В

    • Буква I = ток в амперах, A

    • Буква R = сопротивление нагрузки в схема в омах, Вт

    • Буква r = внутреннее сопротивление ячейки в омах, Вт

    [Изображение будет скоро загружено]

    Здесь преобразование приведенного выше уравнения выглядит следующим образом:

    ε = IR + Ir

    , а затем к следующему:

    ε = V + Ir

    В этом уравнении мы знаем, что появляется буква V, которая обозначает разность потенциалов на клеммах, которая обычно измеряется в вольтах, то есть V.Это разность потенциалов, которая, как говорят, возникает на выводах ячейки, когда в цепи протекает ток, то есть она всегда меньше, чем ЭДС. ячейки.

    Можно сказать, например:

    1. Можно сказать, что п.о. То есть на выводах элемента есть 3,0 вольта, когда он не подключен к цепи и ток не течет. Здесь, когда ячейка, как говорят, подключена к цепи и ток 0,37 А протекает через клемму p.d. Обычно это число падает до 2.8 В. можно сказать, что какое внутреннее сопротивление ячейки?

    График терминала, который имеет p.d. против тока

    Итак, мы можем сказать, что если мы построим график разности выводов потенциалов, обозначенной буквой V, в зависимости от тока в цепи, обозначенного буквой I, мы получим прямую линию с отрицательным градиентом.

    [Изображение скоро будет загружено]

    Тут можно сказать, что им вообще эм.ф. переставляют. уравнение сверху, чтобы соответствовать общему выражению для прямой линии, то есть мы можем сказать y = mx + c.

    [Изображение будет загружено в ближайшее время]

    Мы можем отметить, что это из красных прямоугольников выше:

    • Перехват, который, как предполагается, находится на оси Y, равен e.m.f. То есть ячейки

    • Градиент графика равен -r, где r — внутреннее сопротивление ячейки.

    Можно сказать, что физика чистая Поиск по сайту.

    Внутреннее сопротивление клетки Формула

    Электродвижущая сила, которая представляет собой ЭДС, является незнакомым понятием, которое знакомо большинству студентов.Эти вещи тесно связаны с более знакомым понятием напряжения. Понимание разницы между этими двумя и того, что обычно означает ЭДС, дает нам инструменты, необходимые для решения многих проблем в физике, а также в электронике. Также будет представлена ​​концепция внутреннего сопротивления батареи. Здесь снова мы можем сказать, что ЭДС говорит о напряжении батареи, которое без внутреннего сопротивления, уменьшающего значение. Говорят, что в этой теме объясняется формула ЭДС на примерах.Итак, мы снова видим, что давайте изучим это.

    Электродвижущая сила уже определяется как разность потенциалов на выводах батареи, то есть мы можем сказать, когда через нее не течет ток. Говорят, что это не похоже на это, поскольку это будет иметь значение, но мы можем сказать, что каждая батарея имеет внутреннее сопротивление. Можно сказать, что оно похоже на обычное сопротивление, уменьшающее ток в цепи, но оно существует внутри самой батареи.

    Нет тока, протекающего через элемент, который называется внутренним сопротивлением, которое ничего не изменит, потому что нет тока для его замедления.

    Электродвижущая сила и внутреннее сопротивление

    Электродвижущая сила (э) или э.д.с. это энергия, обеспечиваемая элементом или батареей на один кулон заряда, проходящего через них, она составляет , измеренная в вольтах (В). Он равен разности потенциалов на выводах ячейки, когда ток не течет.

    • e = электродвижущая сила в вольтах, В
    • E = энергия в джоулях, Дж
    • Q = заряд в кулонах, Кл

    Батареи и элементы имеют внутреннее сопротивление (r) , что составляет единиц Ом (Вт). Когда электричество течет по цепи, внутреннее сопротивление самой ячейки сопротивляется прохождению тока, и поэтому тепловая энергия теряется в самой ячейке.

    • e = электродвижущая сила в вольтах, В
    • I = ток в амперах, А
    • R = сопротивление нагрузки в цепи в Ом, Вт
    • r = внутреннее сопротивление ячейки в Ом, Вт

    Мы можем изменить приведенное выше уравнение;

    , а затем на

    В этом уравнении ( В, ) появляется разность потенциалов на клеммах , измеренная в вольтах (В).Это разность потенциалов на выводах ячейки при протекании тока в цепи, она всегда меньше ЭДС. ячейки.

    Пример;

    Q1) p.d. на выводах элемента составляет 3,0 В, когда он не подключен к цепи и не течет ток. Когда ячейка подключена к цепи и течет ток 0,37 А, клемма p.d. падает до 2,8 В. Какое внутреннее сопротивление ячейки?

    График терминала п.d. против текущего

    Если мы построим график разности потенциалов на клеммах (V) в зависимости от тока в цепи (I), мы получим прямую линию с отрицательным градиентом.

    Мы можем им переставить э.д.с. уравнение сверху, чтобы соответствовать общему выражению для прямой линии, y = mx + c.

    Из красных прямоугольников выше видно;

    • пересечение по оси Y равно ЭДС. ячейки
    • градиент графика равен -r, где r — внутреннее сопротивление ячейки.

    ЭДС и внутреннее сопротивление

    ЭДС и внутреннее сопротивление
    следующий: резисторы последовательно и Up: Электрический ток Предыдущий: Сопротивление и удельное сопротивление Теперь настоящие батареи изготавливаются из материалов с ненулевым удельным сопротивлением. Отсюда следует, что настоящие батареи — это не просто источники чистого напряжения. Они также обладают внутренние сопротивления . Между прочим, чистое напряжение Источник обычно упоминается как ЭДС (что означает электродвижущая сила ).Конечно, ЭДС измеряется в вольтах. Батарею можно смоделировать как ЭДС, включенную последовательно с резистором. , который представляет собой его внутреннее сопротивление. Предположим, что такие батарея используется для управления током через внешний нагрузочный резистор, так как показано на рис.17. Обратите внимание, что на принципиальных схемах ЭДС представлена ​​в виде двух близко расположенных параллельных линии неравной длины. Электрический потенциал более длинной линии больше, чем тот из более коротких по вольтам. Резистор представлен как зигзагообразная линия.
    Рисунок 17: Батарея ЭДС и внутреннего сопротивления подключена к нагрузочному резистору сопротивления.

    Рассмотрим батарею на рисунке. Напряжение аккумулятора равно определяется как разница в электрическом потенциале между его положительным и отрицательные клеммы: т.е. , точки и соответственно. Когда мы переходим от к , электрический потенциал увеличивается на вольт, когда мы пересекаем ЭДС, но затем уменьшается на вольт, когда мы пересекаем внутренний резистор.Падение напряжения на резисторе следует из закона Ома, из которого следует, что падение напряжения на резисторе, несущем ток , находится в том направлении, в котором текущие потоки. Таким образом, напряжение аккумулятора связано с его ЭДС. и внутреннее сопротивление через

    (133)

    Обычно мы думаем, что ЭДС батареи по существу постоянная (поскольку она зависит только от химической реакции, происходящей внутри батареи, которая преобразует химическая энергия в электрическую), поэтому мы должны заключить, что напряжение батарея на самом деле уменьшается по мере увеличения тока, потребляемого от нее.Фактически, напряжение равно только ЭДС при пренебрежимо малом токе. Текущий розыгрыш от аккумулятора обычно не может превышать критического значения
    (134)

    поскольку напряжение становится отрицательным (что может произойти только если резистор нагрузки также отрицательный: это практически невозможно). Отсюда следует, что если мы закоротим аккумулятор, подключив его положительные и отрицательные клеммы вместе с использованием проводника с незначительным сопротивлением, ток, потребляемый батареей, ограничен ее внутренним сопротивлением.Фактически в этом случае сила тока равна максимально возможной. Текущий .

    Настоящая батарея обычно характеризуется его ЭДС ( т.е. , его напряжение при нулевом токе) и максимальный ток, который он может подавать. Например, стандартный сухой элемент ( т.е. , своего рода аккумулятор, используемый для питания калькуляторов и фонарей) обычно рассчитан на и скажи) . Таким образом, ничего действительно катастрофического не произойдет. произойдет, если мы закоротим сухой элемент.Мы разрядим аккумулятор через сравнительно короткий промежуток времени, но опасно большой ток не будет поток. С другой стороны, автомобильный аккумулятор обычно рассчитывается на и что-то вроде (такой ток нужен для запустить стартер). Понятно, что автомобильный аккумулятор должен иметь много меньшее внутреннее сопротивление, чем у сухого элемента. Отсюда следует, что если мы были достаточно глупы, чтобы замкнуть автомобильный аккумулятор, в результате довольно катастрофически (представьте себе всю энергию, необходимую для запуска двигателя автомобиль собирается тонким проводом, соединяющим клеммы аккумулятора вместе).



    следующий: резисторы последовательно и Up: Электрический ток Предыдущий: Сопротивление и удельное сопротивление
    Ричард Фицпатрик 2007-07-14

    Оценка внутреннего сопротивления в цепях | Электрические схемы

    Рабочий пример 7: Внутреннее сопротивление в цепи с последовательными резисторами

    Для следующей схемы рассчитайте:

    1. разности потенциалов \ (V_ \ text {1} \), \ (V_ \ text {2} \) и \ (V_ \ text {3} \) на резисторах \ (R_ \ text {1} \), \ (R_ \ text {2} \) и \ (R_ \ text {3} \)

      .
    2. сопротивление \ (R_ \ text {3} \).

    3. сопротивление \ (R_ \ text {3} \).

    Если внутреннее сопротивление равно \ (\ text {0,1} \) \ (\ text {Ω} \), какова ЭДС батареи и какая мощность рассеивается внутренним сопротивлением батареи?

    Примечание

    Это вопрос, очень похожий на то, что вы видели ранее. Это необходимо для того, чтобы выделить Дело в том, что подход к внутреннему сопротивлению строится на том же принципы, с которыми вы уже работали.

    Определите, как подойти к проблеме

    Нам дана разность потенциалов на ячейке и ток в цепи, а также сопротивления двух из трех резисторов. Мы можем использовать закон Ома для расчета разности потенциалов на известных резисторах. Поскольку резисторы включены в последовательную цепь, разность потенциалов равна \ (V = V_ \ text {1} + V_ \ text {2} + V_ \ text {3} \), и мы можем вычислить \ (V_ \ text {3} \). Теперь мы можем использовать эту информацию, чтобы найти разность потенциалов на неизвестном резисторе \ (R_ \ text {3} \).

    Вычислить разность потенциалов на \ (R_ \ text {1} \)

    Использование закона Ома: \ begin {align *} R_ \ text {1} & = \ frac {V_ \ text {1}} {I} \\ I \ cdot R_ \ text {1} & = I \ cdot \ frac {V_ \ text {1}} {I} \\ V_ \ text {1} & = {I} \ cdot {R_ \ text {1}} \\ & = 2 \ cdot 1 \\ V_ \ текст {1} & = \ текст {2} \ текст {V} \ end {align *}

    Вычислить разность потенциалов на \ (R_ \ text {2} \)

    Снова используя закон Ома: \ begin {align *} R_ \ text {2} & = \ frac {V_ \ text {2}} {I} \\ I \ cdot R_ \ text {2} & = I \ cdot \ frac {V_ \ text {2}} {I} \\ V_ \ text {2} & = {I} \ cdot {R_ \ text {2}} \\ & = 2 \ cdot 3 \\ V_ \ текст {2} & = \ текст {6} \ текст {V} \ end {align *}

    Вычислить разность потенциалов на \ (R_ \ text {3} \)

    Поскольку разность потенциалов на всех резисторах, вместе взятых, должна быть такой же, как разность потенциалов на ячейке в последовательной цепи, мы можем найти \ (V_ \ text {3} \), используя: \ begin {align *} V & = V_ \ text {1} + V_ \ text {2} + V_ \ text {3} \\ V_ \ text {3} & = V — V_ \ text {1} — V_ \ text {2} \\ & = 23-2-6 \\ V_ \ текст {3} & = \ текст {15} \ текст {V} \ end {align *}

    Найдите сопротивление \ (R_ \ text {3} \)

    Нам известна разность потенциалов на \ (R_ \ text {3} \) и ток через нее, поэтому мы можем использовать закон Ома для вычисления значения сопротивления: \ begin {align *} R_ \ text {3} & = \ frac {V_ \ text {3}} {I} \\ & = \ frac {\ text {15}} {\ text {2}} \\ R_ \ text {3} & = \ text {7,5} ~ ​​\ Omega \ end {align *}

    Разница потенциалов на внутреннем сопротивлении батареи

    Значение ЭДС можно рассчитать по разности потенциалов нагрузки и разности потенциалов на внутреннем сопротивлении.2} {R} \), и мы знаем ток в цепи, внутреннее сопротивление и разность потенциалов на нем, поэтому мы можем использовать любую форму уравнения для мощности:

    \ begin {align *} P_r & = V_rI_r ​​\\ & = (\ текст {0,2}) (\ текст {2}) \\ & = \ текст {0,4} \ текст {W} \ end {align *}

    Напишите окончательный ответ

    • \ (V_ \ text {1} = \ text {2,0} \ text {V} \)
    • \ (V_ \ text {2} = \ text {6,0} \ text {V} \)
    • \ (V_ \ text {3} = \ text {10,0} \ text {V} \)
    • \ (R_ \ text {3} = \ text {7,5} \ Omega \)
    • \ (\ mathcal {E} = \ text {23,2} \ text {V} \)
    • \ (P_r = \ text {0,4} \ text {W} \)

    Рабочий пример 8: Внутреннее сопротивление и резисторы параллельно

    Разность потенциалов на батарее составляет 18 В, когда она подключена к двум параллельным резисторам \ (\ text {4,00} \) \ (\ Omega \) и \ (\ text {12,00} \) \ ( \ Omega \) соответственно.Рассчитайте ток через ячейку и через каждый из резисторов. Если внутреннее сопротивление батареи \ (\ text {0,375} \) \ (\ text {Ω} \), какова ЭДС батареи?

    Сначала нарисуйте схему перед выполнением любых расчетов

    Определите, как подойти к проблеме

    Нам нужно определить ток через ячейку и каждый из параллельных резисторов. Нам дана разность потенциалов на ячейке и сопротивления резисторов, поэтому мы можем использовать закон Ома для расчета тока.

    Рассчитать ток через ячейку

    Чтобы рассчитать ток через элемент, нам сначала нужно определить эквивалентное сопротивление остальной части цепи. Резисторы включены параллельно и поэтому: \ begin {align *} \ frac {\ text {1}} {R} & = \ frac {\ text {1}} {R_ \ text {1}} + \ frac {\ text {1}} {R_ \ text {2}} \ \ & = \ frac {\ text {1}} {\ text {4}} + \ frac {\ text {1}} {\ text {12}} \\ & = \ frac {3 + 1} {\ text {12}} \\ & = \ frac {\ text {4}} {\ text {12}} \\ R & = \ frac {\ text {12}} {\ text {4}} = \ text {3,00} \ \ Omega \ end {выровнять *} Теперь, используя закон Ома, чтобы найти ток через ячейку: \ begin {align *} R & = \ frac {V} {I} \\ I & = \ frac {V} {R} \\ & = \ frac {\ text {18}} {\ text {3}} \\ I & = \ text {6,00} \ text {A} \ end {align *}

    Теперь определите ток через один из параллельных резисторов

    Мы знаем, что для чисто параллельной конфигурации резисторов разность потенциалов на ячейке такая же, как и разность потенциалов на каждом из параллельных резисторов.Для этой схемы: \ begin {align *} V & = V_ \ text {1} = V_ \ text {2} = \ text {18} \ text {V} \ end {выровнять *} Начнем с вычисления тока через \ (R_ \ text {1} \) по закону Ома: \ begin {align *} R_ \ text {1} & = \ frac {V_ \ text {1}} {I_ \ text {1}} \\ I_ \ text {1} & = \ frac {V_ \ text {1}} {R_ \ text {1}} \\ & = \ frac {\ text {18}} {\ text {4}} \\ I_ \ text {1} & = \ text {4,50} \ text {A} \ end {align *}

    Рассчитайте ток через другой параллельный резистор

    Мы можем снова использовать закон Ома, чтобы найти ток в \ (R_ \ text {2} \): \ begin {align *} R_ \ text {2} & = \ frac {V_ \ text {2}} {I_ \ text {2}} \\ I_ \ text {2} & = \ frac {V_ \ text {2}} {R_ \ text {2}} \\ & = \ frac {\ text {18}} {\ text {12}} \\ I_ \ text {2} & = \ text {1,50} \ text {A} \ end {выровнять *} Альтернативный метод вычисления \ (I_ \ text {2} \) заключался бы в использовании того факта, что токи через каждый из параллельных резисторов должны составлять общий ток через ячейку: \ begin {align *} I & = I_ \ text {1} + I_ \ text {2} \\ I_ \ text {2} & = I — I_ \ text {1} \\ & = 6 — 4.5 \\ I_ \ text {2} & = \ text {1,5} \ text {A} \ end {align *}

    Определить ЭДС

    Суммарный ток через батарею — это ток через внутреннее сопротивление батареи. Знание силы тока и сопротивления позволяет нам использовать закон Ома для определения разности потенциалов на внутреннем сопротивлении и, следовательно, ЭДС батареи.

    Используя закон Ома, мы можем определить разность потенциалов на внутреннем сопротивлении:

    \ begin {align *} V & = I \ cdot r \\ & = \ текст {6} \ cdot \ text {0,375} \\ & = \ текст {2,25} \ текст {V} \ end {выровнять *}

    Мы знаем, что ЭДС аккумулятора — это разность потенциалов на выводе, суммированная с разностью потенциалов на внутреннем сопротивлении, поэтому:

    \ begin {align *} \ mathcal {E} & = V + Ir \\ & = \ text {18} + \ text {2,25} \\ & = \ текст {20,25} \ текст {V} \ end {align *}

    Напишите окончательный ответ

    Ток через ячейку равен \ (\ text {6,00} \) \ (\ text {A} \).

    Ток через резистор \ (\ text {4,00} \) \ (\ Omega \) равен \ (\ text {4,50} \) \ (\ text {A} \).

    Ток через резистор \ (\ text {12,00} \) \ (\ Omega \) равен \ (\ text {1,50} \) \ (\ text {A} \).

    ЭДС батареи равна \ (\ text {20,25} \) \ (\ text {V} \).

    Рабочий пример 9: Питание в последовательной и параллельной сетях резисторов

    Дана следующая схема:

    Ток, покидающий батарею, равен \ (\ text {1,07} \) \ (\ text {A} \), общая мощность, рассеиваемая во внешней цепи, равна \ (\ text {6,42} \) \ ( \ text {W} \), отношение полных сопротивлений двух параллельных сетей \ (R_ {P \ text {1}}: R_ {P \ text {2}} \) равно 1: 2, соотношение \ (R_ \ text {1}: R_ \ text {2} \) равно 3: 5 и \ (R_ \ text {3} = \ text {7,00} \ text {Ω} \).

    Определите:

    1. разность потенциалов АКБ,
    2. мощность, рассеиваемая в \ (R_ {P \ text {1}} \) и \ (R_ {P \ text {2}} \), и
    3. , если батарея обозначена как имеющая ЭДС \ (\ text {6,50} \) \ (\ text {V} \), каково значение сопротивления каждого резистора и мощность, рассеиваемая в каждом из них.

    Что требуется

    В этом вопросе вам предоставляется различная информация и предлагается определить мощность, рассеиваемую на каждом резисторе и каждой комбинации резисторов.Обратите внимание, что данная информация в основном относится ко всей цепи. Это подсказка, которую вы должны начать с общей схемы и двигаться вниз к более конкретным элементам схемы.

    Расчет разности потенциалов аккумулятора

    В первую очередь остановимся на аккумуляторе. Нам дана мощность всей цепи, а также ток, покидающий батарею. Мы знаем, что разность потенциалов на клеммах аккумулятора — это разность потенциалов в цепи в целом.

    Мы можем использовать соотношение \ (P = VI \) для всей схемы, потому что разность потенциалов такая же, как разность потенциалов на клеммах батареи: \ begin {align *} P & = VI \\ V & = \ frac {P} {I} \\ & = \ frac {\ text {6,42}} {\ text {1,07}} \\ & = \ текст {6,00} \ текст {V} \ end {align *}

    Разность потенциалов на батарее равна \ (\ text {6,00} \) \ (\ text {V} \).

    Мощность, рассеиваемая в \ (R_ {P \ text {1}} \) и \ (R_ {P \ text {2}} \)

    Помните, что мы работаем от общих деталей схемы вниз к деталям отдельных элементов, это противоположно тому, как вы относились к этой схеме ранее.

    Мы можем рассматривать параллельные сети как эквивалентные резисторы, поэтому схема, с которой мы сейчас работаем, выглядит так:

    Мы знаем, что ток через два элемента схемы будет одинаковым, потому что это последовательная цепь и что сопротивление всей цепи должно быть: \ (R_ {Ext} = R_ {P \ text {1}} + R_ {P \ text {2}} \). Мы можем определить полное сопротивление по закону Ома для цепи в целом: \ begin {align *} V_ {батарея} & = IR_ {Ext} \\ R_ {Ext} & = \ frac {V_ {аккумулятор}} {I} \\ & = \ frac {\ text {6,00}} {\ text {1,07}} \\ & = \ текст {5,61} \ текст {Ω} \ end {align *}

    Мы знаем, что соотношение между \ (R_ {P \ text {1}}: R_ {P \ text {2}} \) равно 1: 2, что означает, что мы знаем: \ begin {align *} R_ {P \ text {1}} & = \ frac {\ text {1}} {\ text {2}} R_ {P \ text {2}} \ \ \ text {и} \\ R_T & = R_ {P \ text {1}} + R_ {P \ text {2}} \\ & = \ frac {\ text {1}} {\ text {2}} R_ {P \ text {2}} + R_ {P \ text {2}} \\ & = \ frac {\ text {3}} {\ text {2}} R_ {P \ text {2}} \\ (\ text {5,61}) & = \ frac {\ text {3}} {\ text {2}} R_ {P \ text {2}} \\ R_ {P \ text {2}} & = \ frac {\ text {2}} {\ text {3}} (\ text {5,61}) \\ R_ {P \ text {2}} & = \ text {3,74} \ text {Ω} \ end {выровнять *} и поэтому: \ begin {align *} R_ {P \ text {1}} & = \ frac {\ text {1}} {\ text {2}} R_ {P \ text {2}} \\ & = \ frac {\ text {1}} {\ text {2}} (3.2 (\ text {3,74}) \\ & = \ текст {4,28} \ текст {W} \ end {выровнять *} Эти значения будут в сумме с исходным значением мощности, которое у нас было для внешней цепи. Если бы они не мы бы сделали ошибку в расчетах.

    Расчет параллельной сети 1

    Теперь мы можем приступить к детальному расчету первого набора параллельных резисторов.

    Мы знаем, что соотношение между \ (R _ {\ text {1}}: R _ {\ text {2}} \) составляет 3: 5, что означает, что мы знаем \ (R _ {\ text {1}} = \ frac {\ text {3}} {\ text {5}} R _ {\ text {2}} \).Нам также известно общее сопротивление двух параллельных резисторов в этой сети. это \ (\ text {1,87} \) \ (\ text {Ω} \). Мы можем использовать соотношение между значениями двух резисторов, а также формула для общей сопротивление (\ (\ frac {\ text {1}} {R_PT} = \ frac {\ text {1}} {R_ \ text {1}} + \ frac {\ text {1}} {R_ \ text {2 }} \)) чтобы найти номиналы резисторов: \ begin {align *} \ frac {\ text {1}} {R_ {P \ text {1}}} & = \ frac {\ text {1}} {R_ \ text {1}} + \ frac {\ text {1}} { R_ \ text {2}} \\ \ frac {\ text {1}} {R_ {P \ text {1}}} & = \ frac {\ text {5}} {3R_ \ text {2}} + \ frac {\ text {1}} { R_ \ text {2}} \\ \ frac {\ text {1}} {R_ {P \ text {1}}} & = \ frac {\ text {1}} {R_ \ text {2}} (\ frac {\ text {5}} { \ text {3}} + 1) \\ \ frac {\ text {1}} {R_ {P \ text {1}}} & = \ frac {\ text {1}} {R_ \ text {2}} (\ frac {\ text {5}} { \ text {3}} + \ frac {\ text {3}} {\ text {3}}) \\ \ frac {\ text {1}} {R_ {P \ text {1}}} & = \ frac {\ text {1}} {R_ \ text {2}} \ frac {\ text {8}} {\ текст {3}} \\ R_ \ text {2} & = R_ {P \ text {1}} \ frac {\ text {8}} {\ text {3}} \\ & = (\ text {1,87}) \ frac {\ text {8}} {\ text {3}} \\ & = \ текст {4,99} \ текст {Ω} \ end {выровнять *} Мы также можем вычислить \ (R _ {\ text {1}} \): \ begin {align *} R _ {\ text {1}} & = \ frac {\ text {3}} {\ text {5}} R _ {\ text {2}} \\ & = \ frac {\ text {3}} {\ text {5}} (\ text {4,99}) \\ & = \ текст {2,99} \ текст {Ω} \ end {align *}

    Для определения мощности нам нужно рассчитанное нами сопротивление и либо разность потенциалов, либо ток. 2} {\ text {4,99}} \\ & = \ текст {0,80} \ текст {W} \ end {align *}

    Параллельная сеть 2 расчета

    Теперь мы можем приступить к детальному расчету второго набора параллельных резисторов.

    Нам дан \ (R_ \ text {3} = \ text {7,00} \ text {Ω} \), и мы знаем \ (R_ {P \ text {2}} \), поэтому мы можем вычислить \ (R_ \ text {4} \) из: \ begin {align *} \ frac {\ text {1}} {R_ {P \ text {2}}} & = \ frac {\ text {1}} {R_ \ text {3}} + \ frac {\ text {1}} { R_ \ text {4}} \\ \ frac {\ text {1}} {\ text {3,74}} & = \ frac {\ text {1}} {\ text {7,00}} + \ frac {\ text {1}} {R_ \ текст {4}} \\ R_ \ text {4} & = \ text {8,03} \ text {Ω} \ end {align *}

    Мы можем вычислить разность потенциалов во второй параллельной сети, вычтя разность потенциалов первой параллельной сети из разности потенциалов батареи, \ (V_ {P \ text {2}} = \ text {6,00} — \ text {2,00} = \ text {4,00} \ text {V} \).2} {\ text {8,03}} \\ & = \ текст {1,99} \ текст {W} \ end {align *}

    Внутреннее сопротивление

    Мы знаем, что ЭДС батареи равна \ (\ text {6,5} \) \ (\ text {V} \), но что разность потенциалов, измеренная на клеммах, составляет всего \ (\ text {6} \) \ (\ текст {V} \). Разница — это разность потенциалов на внутреннем сопротивлении батареи, и мы можем использовать известный ток и закон Ома для определения внутреннего сопротивления:

    \ begin {align *} V & = I \ cdot R \\ R & = \ frac {V} {I} \\ & = \ frac {\ text {0,5}} {\ text {1,07}} \\ & = \ text {0,4672897} \\ & = \ текст {0,47} \ текст {Ω} \ end {выровнять *}

    Мощность, рассеиваемая внутренним сопротивлением батареи:

    \ begin {align *} P & = VI \\ & = \ текст {0,5} \ cdot \ text {1,07} \\ & = \ текст {0,535} \ текст {W} \ end {align *}

    Рабочий пример 10: Внутреннее сопротивление и фары [NSC 2011 Paper 1]

    Фара и два ИДЕНТИЧНЫХ задних фонаря скутера подключены параллельно к батарее с неизвестным внутренним сопротивлением, как показано на упрощенной принципиальной схеме ниже.Фара имеет сопротивление \ (\ text {2,4} \) \ (\ text {Ω} \) и управляется переключателем \ (\ textbf {S} _1 \). Задние фонари управляются переключателем \ (\ textbf {S} _2 \). Сопротивлением соединительных проводов можно пренебречь.

    На приведенном рядом графике показана разность потенциалов на клеммах батареи до и после включения переключателя \ (\ textbf {S} _1 \) (пока переключатель \ (\ textbf {S} _2 \) открыт). Переключатель \ (\ textbf {S} _1 \) закрывается в момент \ (\ textbf {t} _1 \).

    1. Используйте график, чтобы определить ЭДС аккумулятора.

      (1 балл)

    2. ПРИ ТОЛЬКО ПЕРЕКЛЮЧАТЕЛЬ \ (\ textbf {S} _1 \) ЗАКРЫТ, вычислите следующее:

      1. Ток через фару

        (3 балла)

      2. Внутреннее сопротивление \ (r \) батареи

        (3 балла)

    3. ОБЕ ПЕРЕКЛЮЧАТЕЛИ \ (\ textbf {S} _1 \) И \ (\ textbf {S} _2 \) ТЕПЕРЬ ЗАКРЫТЫ.В течение этого периода аккумулятор обеспечивает ток \ (\ text {6} \) \ (\ text {A} \).

      Рассчитайте сопротивление каждого заднего фонаря.

      (5 баллов)

    4. Как повлияет на показания вольтметра, если фара перегорит? (Оба переключателя \ (\ textbf {S} _1 \) и \ (\ textbf {S} _2 \) все еще закрыты.)

      Запишите только УВЕЛИЧИВАЕТ, УМЕНЬШАЕТСЯ или ОСТАЕТСЯ ОДИН ТО ЖЕ.

      Дайте объяснение.

      (3 балла)

    Вопрос 1

    \ (\ text {12} \) \ (\ text {V} \)

    (1 балл)

    Вопрос 2.1

    Вариант 1:

    \ begin {align *} I & = \ frac {V} {R} \\ & = \ frac {\ text {9,6}} {\ text {2,4}} \\ & = \ текст {4 A} \ end {выровнять *}

    Вариант 2:

    \ begin {align *} \ text {emf} & = IR + Ir \\ 12 & = I (\ text {2,4}) + \ text {2,4} \\ \ поэтому I & = \ text {4 A} \ end {выровнять *}

    (3 балла)

    Вопрос 2.2

    Вариант 1:

    \ begin {align *} \ text {emf} & = IR + Ir \\ 12 & = \ text {9,4} + 4r \\ r & = \ текст {0,6} \ \ Omega \ end {выровнять *}

    Вариант 2:

    \ begin {align *} V_ {потеряно} & = Ir \\ \ text {2,4} & = \ text {4} r \\ \ поэтому r & = \ text {0,6} \ \ Omega \ end {выровнять *}

    Вариант 3:

    \ begin {align *} \ text {emf} & = I (R + r) \\ \ text {12} & = \ text {4} (\ text {2,4} + r) \\ \ поэтому r & = \ text {0,6} \ \ Omega \ end {выровнять *}

    (3 балла)

    Вопрос 3

    Вариант 1:

    \ begin {align *} \ text {emf} & = IR + Ir \\ \ text {12} & = \ text {6} (R + \ text {0,6}) \\ R _ {\ text {ext}} & = \ text {1,4} \ \ Omega \ конец {выравнивание *} \ begin {выравнивание *} \ frac {1} {R} & = \ frac {1} {R_ {1}} + \ frac {1} {R_ {2}} \\ \ frac {1} {\ text {1,4}} & = \ frac {1} {\ text {2,4}} + \ frac {1} {R} \\ R & = \ текст {3,36} \ \ Omega \ end {выровнять *}

    Каждый задний фонарь: \ (R = \ text {1,68} \ \ Omega \)

    Вариант 2:

    \ begin {align *} \ text {Emf} & = V _ {\ text {terminal}} + Ir \\ 12 & = V _ {\ text {терминал}} + 6 (\ text {0,6}) \\ \ поэтому V _ {\ text {terminal}} & = \ text {8,4} \ text {V} \ конец {выравнивание *} \ begin {выравнивание *} I _ {\ text {2,4} \ \ Omega} & = \ frac {V} {R} \\ & = \ frac {\ text {8,4}} {\ text {2,4}} \\ & = \ text {3,5 A} \ конец {выравнивание *} \ begin {выравнивание *} I _ {\ text {задние фонари}} & = 6 — \ text {3,5} \\ & = \ текст {2,5} \ текст {A} \\ R _ {\ text {задние фонари}} & = \ frac {V} {I} \\ & = \ frac {\ text {8,4}} {\ text {2,5}} \\ & = \ текст {3,36} \ \ Omega \\ R _ {\ text {задний фонарь}} & = \ text {1,68} \ \ Omega \ end {выровнять *}

    Вариант 3:

    \ begin {align *} V & = IR \\ \ text {12} & = \ text {6} (R) \\ R _ {\ text {ext}} & = 2 \ \ Omega \ конец {выравнивание *} \ begin {выравнивание *} R _ {\ text {parallel}} & = 2 — \ text {0,6} \\ & = \ текст {1,4} \ \ Omega \\ \ frac {1} {R} & = \ frac {1} {R_ {1}} + \ frac {1} {R_ {2}} \\ \ frac {1} {\ text {1,4}} & = \ frac {1} {\ text {2,4}} + \ frac {1} {R} \\ R & = \ текст {3,36} \ \ Omega \ end {выровнять *}

    Каждый задний фонарь: \ (R = \ text {1,68} \ \ Omega \)

    Вариант 4:

    Для параллельной комбинации: \ (I_ {1} + I_ {2} = 6 \ text {A} \)

    \ begin {align *} \ поэтому \ frac {V} {\ text {2,4}} + \ frac {V} {R _ {\ text {задние фонари}}} & = \ text {6} \\ \ text {8,4} \ left (\ frac {1} {\ text {2,4}} + \ frac {1} {R _ {\ text {задние фонари}}} \ right) & = \ text {6 } \\ \ поэтому R _ {\ text {задние фонари}} & = \ text {3,36} \ \ Omega \\ R _ {\ text {задний фонарь}} & = \ text {1,68} \ \ Omega \ end {выровнять *}

    (5 баллов)

    Вопрос 4

    Увеличивается

    Сопротивление увеличивается, а ток уменьшается.Таким образом, \ (Ir \) (потерянное вольт) должно уменьшаться, что приводит к увеличению напряжения.

    (3 балла)

    [ИТОГО: 15 баллов]

    Cells EMF Internal Resistance — Учебный материал для IIT JEE


    История

    Луиджи Гальвани Итальянский ученый, отрезая лягушачью лапу, его стальной скальпель задел один из нервов лягушки, и мышцы ноги дернулись. Нога дернулась. Он назвал это явление феноменом животным электричеством.Гальвани тогда считал, что это электричество является врожденным или естественным для живых существ, включая человека. Затем он был удостоен чести за открытие биоэлектричества. Но позже Алессандро Вольта не согласился с его открытием. Он проделал тот же эксперимент и смог воспроизвести тот же результат. Хотя Вольта получил такой же результат, заявление Гальвани его не удовлетворило. Вольта сказал, что ноги дергались из-за двух разных металлов.

    Volta затем провел еще один эксперимент, чтобы доказать это.Он приготовил стопку чередующихся слоев меди и цинка, которые были разделены бумагой или тканью, смоченной в смеси с соленой водой. В этом эксперименте цинк теряет электроны, и, таким образом, происходит реакция окисления. Эти электроны затем принимаются ионами в воде и, таким образом, происходит реакция восстановления. Это основа элемента или батареи. Алессандро Вольта изобрел электрическую батарею. Впервые он был назван Voltaic Pile. За его вклад в науку единица электрического потенциала названа Вольт .

    Гальваническая свая

    Позже ученые улучшили дизайн Вольты. Гальваническая батарея не могла обеспечивать ток в течение более длительных периодов времени. Затем John Frederic Daniell разработал ячейку Daniell. Таким образом он избежал проблем с коррозией изобретения Вольта. Затем Джордж Лекланш изобрел мокрый элемент, а доктор Карл Гасснер представил сухую батарею. Ученые заменили химический раствор сухими ячейками, заполненными химической пастой.Здесь металл снова окисляется, и происходят реакции как окисления, так и восстановления. Электрический ток не был полностью изучен до того, как были разработаны батареи. Сухие батареи — один из наиболее часто используемых типов батарей в наши дни.

    Простая схема сухого элемента

    Gaston Plante представил первую аккумуляторную батарею. Это свинцово-кислотная аккумуляторная батарея, которая снова наиболее часто используется в автомобилях.


    Ячейка

    Мы знаем, что электрический ток — это поток заряженных частиц.Это поток электронов по цепи.

    Ячейка — это устройство, которое поддерживает разность потенциалов, которая существует между двумя электродами из-за химической реакции. Набор из двух или более ячеек, которые соединены последовательно или параллельно, называется батареей . Таким образом мы получим необходимое напряжение или ток. Батарея — это источник энергии, преобразующий химическую энергию в электрическую. Он также известен как электрохимическая ячейка. Энергия хранится в форме химической энергии внутри батареи.Аккумуляторы дают нам удобный источник энергии для питания устройств без кабелей и проводов. Когда он подключен к цепи, он производит электрическую энергию.

    Батарея состоит из двух клемм — положительной и отрицательной. Положительный вывод называется катодом , а отрицательный вывод — анодом . Их также называют электродами ячейки . Эти электроды будут погружены в раствор, называемый электролитом. Это жидкость, которая является ионной и проводит электричество.Выходное напряжение батареи зависит от элементов, используемых в качестве электродов, размера электродов и типа используемого в ней электролита.

    Когда аккумулятор собирается заряжаться, к нему подключается внешний источник. Анод батареи подключен к отрицательной клемме источника, а катод — к положительной клемме источника. Поскольку внешний источник подключен к батарее, электроны вставляются в анод. Когда элемент или батарея подключены к цепи, происходят химические реакции.Таким образом, внутри двух электродов происходят химические реакции. Здесь происходят реакции окисления и восстановления. Затем на катоде происходит реакция восстановления, а на аноде — процесс окисления.

    Компоненты ячейки

    Катод действует как окислитель, принимая электроны от отрицательного концевого анода. Анод действует как восстановитель, теряя электроны. Таким образом, из-за этих химических реакций возникает электрическая разница между выводами-анодом и катодом.Когда нет питания, электролит запрещает движение электронов непосредственно от анода к катоду. Вот почему мы используем внешний источник или подключаемся к цепи. Таким образом, электроны перемещаются от анода к катоду, когда цепь замкнута. Наконец, он дает питание подключенному к нему прибору. Спустя долгое время, когда электрохимический процесс изменяет материалы анода и катода, он перестает выделять электроны. Потом садится аккум.


    Первичная ячейка и вторичная ячейка

    Элемент или батарею можно разделить на несколько категорий, таких как первичный элемент или батарея, вторичный элемент или батарея, резервный элемент и топливный элемент.Первичный элемент нельзя зарядить повторно, поэтому его можно использовать только один раз. Химическая реакция будет необратимой, и активные материалы могут не вернуться к своей первоначальной форме в первичной ячейке. Сухие элементы, щелочные элементы и ртутные элементы являются различными примерами первичных элементов. Первичные элементы стоят недорого и могут быть легко использованы. Не подходит для больших нагрузок.

    Вторичные элементы можно заряжать снова и использовать снова и снова. Химическая реакция во вторичной ячейке обратима.Свинцово-кислотный элемент и топливные элементы являются примерами вторичных элементов. Свинцово-кислотный элемент широко используется в транспортных средствах и других приложениях, где требуется большой ток нагрузки. Автомобильные аккумуляторы и резервные источники питания являются вторичными элементами. Солнечные элементы — это вторичные элементы, которые преобразуют энергию солнечного света в электрическую. Хотя стоимость вторичной ячейки выше по сравнению с первичной ячейкой, первичная ячейка может использоваться в течение длительного периода времени. Использование вторичной ячейки более сложное.


    Соединения ячейки

    Элементы могут быть соединены в цепь как последовательно, так и параллельно.Для ячеек, соединенных последовательно, он дает большее результирующее напряжение. Поврежденные элементы можно легко идентифицировать, и, следовательно, их можно легко заменить, поскольку они разрывают цепь. Если какая-либо из ячеек повреждена в цепи, это может повлиять на все соединение. Ячейки, которые соединены последовательно, быстро истощаются, и поэтому они не служат дольше. В домашней электропроводке не используется.

    Если ячейки соединены параллельно и одна из ячеек повреждена в цепи, это не повлияет на все соединение.Ячейки, соединенные параллельно, не изнашиваются легко и, следовательно, служат дольше. Напряжение, развиваемое ячейками при параллельном соединении, не может быть увеличено за счет увеличения количества ячеек, присутствующих в цепи. Это потому, что они не имеют одинаковой круговой траектории. При параллельном подключении подключение обеспечивает питание из расчета на одну ячейку. Так что яркость лампочки не будет высокой.


    Самодельные аккумуляторы

    Из картошки можно сделать в домашних условиях батарею.Для этого эксперимента нам понадобятся картофель, оцинкованный гвоздь, медная монета, два зажима из крокодиловой кожи и вольтметр. Оцинкованные гвозди имеют цинковое покрытие. Возьмите свежий картофель, потому что от его сока зависит результат эксперимента. Вставьте гальванизированный гвоздь в картофель немного по центру. Затем вставьте медную монету в картошку, которая должна быть немного дальше от оцинкованного гвоздя. Теперь к медной монете подключают один зажим к медной монете и вольтметр. Второй зажим подключается к оцинкованному гвоздю, а следующий вывод подключается к вольтметру.Затем проверьте показания вольтметра. Будет небольшое повышение напряжения.

    Картофель в качестве аккумулятора

    Картофельный сок действует как электролит в эксперименте, а цинк в ногте вступает в реакцию с медью. Таким образом происходит химическая реакция. Мы можем привести часы в действие, используя картофельные батарейки. Также можно зажечь лампочки от картофельного аккумулятора. Мы также можем вырабатывать электричество, используя лимонную батарею.

    ЭДС

    ЭДС или электродвижущая сила определяется как разность потенциалов, которая возникает между двумя выводами батареи в разомкнутой цепи.Мы знаем, что анод имеет положительный потенциал (V + ), а катод — отрицательный потенциал (V ). Таким образом, ЭДС — это разность потенциалов между анодом положительного вывода и катодом отрицательного вывода, когда через него не протекает ток. ЭДС измеряет энергию, которая передается заряду, переносимому в элементе или батарее. Это энергия в джоулях, деленная на заряд в кулонах. ЭДС действует как инициирующая сила для протекания тока.

    ε = E / Q, где ε — электродвижущая сила, E — энергия, а Q — заряд.

    ЭДС, которая обозначается ε, а уравнение задается как ε = V + — (-V ) = V + + V -. Измеряется в вольтах.


    Внутреннее сопротивление

    Внутреннее сопротивление — это сопротивление внутри батареи, которое препятствует прохождению тока при подключении к цепи. Таким образом, он вызывает падение напряжения, когда через него протекает ток. Это сопротивление, обеспечиваемое электролитом и электродами, присутствующими в элементе.Таким образом, внутреннее сопротивление обеспечивается электродами и электролитом, которые препятствуют прохождению тока внутри ячейки.


    ЭДС и внутреннее сопротивление

    Рассмотрим схему, приведенную ниже. Ячейку можно модифицировать с помощью ЭДС ε и внутреннего резистора с сопротивлением r, включенного последовательно. Внешний нагрузочный резистор с сопротивлением R также подключен к цепи. Разность потенциалов клемм, представленная как V, определяется как разность потенциалов между положительной и отрицательной клеммами ячейки, когда ток течет по цепи.

    ЭДС и внутреннее сопротивление

    V = V + + V — Ir. Это падение напряжения, вызванное внутренним сопротивлением.

    Мы знаем, что ε = V + + V -. = Я (R + r).

    ε = ИК + Ir.

    = V + Ir

    В = ε — Ir.

    Итак, V = ε — Ir, где V — разность потенциалов в цепи, ε — ЭДС, I — ток, протекающий по цепи, r — внутреннее сопротивление.

    Обычно внутреннее сопротивление ячейки не учитывается, потому что ε >> Ir. Величина внутреннего сопротивления меняется от ячейки к ячейке.

    Эквивалентная ЭДС n ячеек в последовательной комбинации является суммой их индивидуальных ЭДС. Эквивалентное внутреннее сопротивление n ячеек в последовательной комбинации является суммой их индивидуального внутреннего сопротивления. Для n количества ячеек, соединенных параллельно с ЭДС ε 1, ε 2 …… ε n и внутренним сопротивлением r 1 , r 2…. r n

    Сводка
    • Алессандро Вольта изобрел электрическую батарею, которая сначала была названа Voltaic Pile .

    • Ячейка — это ячейка, которая поддерживает разность потенциалов, существующую между двумя электродами из-за химической реакции. Когда две или более ячеек объединяются вместе, они превращаются в батарею. Он преобразует химическую энергию в электрическую.

    • Батарея состоит из двух клемм, которые являются положительной и отрицательной клеммами и также известны как электроды элемента . Положительный вывод называется катодом , а отрицательный вывод называется анодом . Эти электроды будут погружены в раствор, называемый электролитом. Когда происходит химическая реакция, на аноде происходит окисление, а на катоде — восстановление.

    • Элемент или батарея делятся на две основные категории, такие как первичный элемент или батарея и вторичный элемент.Первичная ячейка используется только один раз, а вторичная ячейка может использоваться более одного раза.

    • Элементы можно соединять последовательно и параллельно в цепь.

    • Мы можем сделать батарейки в домашних условиях из картофеля, лимона и т. Д.

    • ЭДС или электродвижущая сила — это разность потенциалов, возникающая между двумя выводами батареи в разомкнутой цепи. ε = E / Q, ε — электродвижущая сила, E — энергия, Q — заряд.

    • Внутреннее сопротивление, которое существует в батарее, препятствует протеканию тока при подключении к цепи.


    Посмотрите это видео, чтобы получить дополнительную информацию


    Дополнительная информация

    Ячейки, ЭДС, внутреннее сопротивление

    Внутреннее сопротивление

    Любой источник электрической энергии, такой как аккумулятор или генератор, поставляет электроэнергию путем преобразования некоторой другой формы энергии в электрическую.В случае с батареей используется энергия, создаваемая какой-либо химической реакцией. Батарея обеспечивает электродвижущую силу (ЭДС), которая толкает электроны вокруг любой электрической цепи, к которой она подключена. Эта электродвижущая сила может быть выражена как напряжение и определяется как общее количество энергии (в джоулях) на единичного заряда (в кулонах), подаваемого в цепь. Его можно представить с помощью следующей формулы:

    куда:

    E = энергия в джоулях

    Q = заряд в кулонах

    ℰ = электродвижущая сила

    Разность потенциалов (pd) на каждом компоненте в цепи также выражается как напряжение, но определяется как энергия на единицу заряда, преобразованная компонентом в другие формы энергии.Полная ЭДС, обеспечиваемая аккумулятором, по существу представляет собой напряжение разомкнутой цепи, аккумуляторной батареи, то есть напряжение, измеренное, когда к аккумуляторной батарее не подключена нагрузка и не протекает ток. На практике это можно измерить достаточно точно с помощью современного цифрового мультиметра, поскольку величина тока, потребляемого мультиметром, может считаться незначительной. В действительности разность потенциалов, измеренная на клеммах батареи, когда она подключена к нагрузке, будет меньше, чем ее напряжение холостого хода.Причина этого в том, что аккумулятор не является идеальным источником напряжения , потому что, помимо протекания тока через нагрузку, аккумулятор также должен пропускать ток через собственное внутреннее сопротивление , что приведет к рассеиванию энергии в виде тепла. .

    Внутреннее сопротивление химической батареи обычно составляет от долей Ом до нескольких Ом и в основном связано с сопротивлением электролитических материалов, используемых при производстве батареи.Ток должен течь между электродами батареи и через эти материалы, когда батарея подключена к электрической цепи. Таким образом, источник напряжения, такой как батарея, можно рассматривать как идеальный источник напряжения (без внутреннего сопротивления), соединенный последовательно с резистором (внутреннее сопротивление батареи). Когда через батарею протекает ток, на внутреннем сопротивлении будет небольшое падение напряжения. Это падение напряжения можно рассчитать как умножение силы тока на внутреннее сопротивление (закон Ома).Когда к источнику напряжения приложена нагрузка, падение напряжения на сопротивлении нагрузки (напряжение на клеммах ) будет равно ЭДС источника минус падение напряжения на внутреннем сопротивлении, поскольку два сопротивления включены последовательно с друг друга. Схема ниже иллюстрирует принцип.


    Внутреннее сопротивление батареи последовательно с сопротивлением нагрузки.


    Внутреннее сопротивление батареи зависит от типа батареи (т.е.е. щелочной , свинцово-кислотный , никель-кадмиевый и т. Д.) И может изменяться в зависимости от нагрузки, температуры и возраста батареи. Одноразовые батареи со временем подают меньшее напряжение, например, потому что их внутреннее сопротивление постоянно увеличивается. В конце концов, вырабатываемое напряжение будет настолько низким, что аккумулятор практически не будет использоваться, и его придется выбросить. Если ЭДС () батареи известна, ее внутреннее сопротивление ( R ВНУТРЕННЯЯ ) можно определить путем измерения тока ( I НАГРУЗКА ), протекающего через сопротивление нагрузки ( R НАГРУЗКА ) с известным значением.Поскольку внутреннее сопротивление и сопротивление нагрузки включены последовательно, мы можем использовать закон Кирхгофа по напряжению и закон Ома, чтобы получить следующую формулу:

    = ( R ВНУТРЕННИЙ + R НАГРУЗКА ) × I НАГРУЗКА

    Решая для R ВНУТРЕННЯЯ , получаем:

    Рассмотрим схему, показанную ниже, в которой батарея с известной ЭДС 1.5 В последовательно с лампой. Падение напряжения, измеренное на лампе В НАГРУЗКА , составляет 1,2 В, что означает, что мы «потеряли» 0,3 В на внутреннем сопротивлении R ВНУТРЕННИЙ . Сопротивление проводов в цепи можно считать незначительным, тогда как сопротивление лампы R НАГРУЗКА неизвестно. Ток I , протекающий по цепи, измеряется как 0,30 А. Как определить внутреннее сопротивление?


    Какое внутреннее сопротивление батареи?


    Поскольку мы знаем, что R = В / (закон Ома), мы можем найти сопротивление нагрузки следующим образом:

    R ВНУТРЕННИЙ = R НАГРУЗКА
    I НАГРУЗКА
    R НАГРУЗКА = V НАГРУЗКА = 1.2 В = 4 Ом
    I 0,3 A

    Теперь, используя полученную ранее формулу для внутреннего сопротивления:

    902 902 902 9019 902 902 902 9019 902 902 902 1.5 В
    R ВНУТРЕННЯЯ = R НАГРУЗКА
    I НАГРУЗКА
    — 4 Ом = 1 Ом
    0,3 А

    Внутреннее сопротивление источника электрической энергии является важным фактором при рассмотрении вопроса о том, как получить от источника максимальную мощность для подключенного к нему электрического прибора (нагрузки). Хотя мы рассмотрим тему мощности более подробно в другом месте, теорема о максимальной мощности утверждает, что максимальная передача мощности происходит, когда внутреннее сопротивление источника равно сопротивлению нагрузки.Обратной стороной этого является то, что мощность, рассеиваемая в самом источнике, также равна мощности, передаваемой нагрузке ( Мощность = I 2 × R ), что дает энергоэффективность всего 50%. Наиболее эффективная передача мощности происходит, когда внешнее (нагрузочное) сопротивление намного превышает внутреннее сопротивление источника. Поэтому при выборе наилучшего типа источника необходимо тщательно учитывать потребности приложения.Например, свинцово-кислотный автомобильный аккумулятор должен обеспечивать относительно высокие токи при сравнительно низком напряжении (большинство автомобильных аккумуляторов вырабатывают номинальную ЭДС 12,6 вольт). Его низкое внутреннее сопротивление позволяет ему обеспечивать такие высокие токи без значительного падения напряжения на клеммах. С другой стороны, высоковольтные источники питания должны иметь чрезвычайно высокое внутреннее сопротивление, чтобы ограничивать ток, который может протекать в случае случайного короткого замыкания.

    Проблемы

    1. Батарея на 9 В подключена последовательно к нагрузке, и напряжение на клеммах равно 8 В.Измеренный ток в цепи составляет 5 А. Какое внутреннее сопротивление батареи?
    2. Какая ЭДС аккумулятора в следующей цепи?


    3. Какое внутреннее сопротивление батареи в следующей цепи?


    4. Аккумулятор имеет внутреннее сопротивление 0,5 Ом и ЭДС 1.5 В. При последовательном подключении к сопротивлению нагрузки напряжение на клеммах падает до 1,45 В. Какой ток течет в цепи и каково значение сопротивления нагрузки?
    5. Каково напряжение на клеммах аккумулятора с ЭДС 12 В и внутренним сопротивлением 0,5 Ом при последовательном подключении к резистору 10 Ом?

    Ответы на проблемы


    Внутреннее сопротивление, ЭДС и разность потенциалов

    В любой цепи есть компоненты, которые передают энергию из в схему, и компоненты, которые отбирают энергию из .С этого момента мы будем говорить, что любое устройство, передающее энергию в цепь, обеспечивает электродвижущую силу (ЭДС) , и любое устройство, выводящее ее, имеет разность потенциалов (pd) на нем.

    И ЭДС, и pd измеряются в вольт, В, , поскольку они описывают, сколько энергии вводится или снимается на кулон заряда, проходящего через этот участок цепи.

    Лучше всего думать о них:

    ЭДС — это количество энергии любой формы, которое превращается в электрическую энергию на кулон заряда.

    pd — количество электрической энергии, которая преобразуется в другие формы энергии на один кулон заряда.

    Источники ЭДС:

    Элемент, аккумулятор (комбинация элементов), солнечный элемент, генератор, динамо-машина, термопара.

    Элементы и батареи не идеальны (что, конечно, не считая момента окончания вашего последнего экзамена?). Используйте их некоторое время, и вы заметите, что они нагреваются.

    Откуда поступает тепловая энергия?

    Это от тока, проходящего через внутреннюю часть ячейки.Сопротивление внутри ячейки превращает часть производимой ею электроэнергии в тепловую при движении электронов через нее.

    Это легко объяснить, если вы вообразите, что каждая ячейка идеальна, за исключением того, что по какой-то странной причине (вероятно, часть заговора по захвату мира, задуманного доктором Злом) производители включили резистор последовательно с ячейкой внутри кожух.

    Следовательно, внутри элемента энергия передается в цепь элементом (ЭДС), но некоторая часть этой энергии забирается из цепи внутренним резистором (pd).

    Таким образом, pd, доступный для остальной части схемы (внешней цепи, поскольку некоторые вопросы могут относиться к ней), это ЭДС минус pd, потерянный внутри ячейки:

    V = E — Ir

    Где :

    В = pd по внешней цепи (В)

    E = ЭДС ячейки (В)

    I = ток через элемент (A)

    r = значение внутреннего сопротивления (& Omega;)

    ( Ir = p.d. на внутреннем резисторе)

    Примечание: В иногда называют клеммой pd , поскольку это pd на клеммах ячейки

    Пример 1:

    Что такое клемма p.d. для ячейки ЭДС 2В и внутреннего сопротивления 1 Ом при подключении к резистору 9 Ом?

    Ответ:

    Просто представьте, что внутреннее сопротивление является одним из нормальных резисторов в цепи.Нарисуйте его на принципиальной схеме рядом с ячейкой, чтобы весь ток, проходящий через ячейку, также проходил через резистор.

    Чтобы найти V, клемму pd (или напряжение, доступное для внешней цепи), рассчитайте ток I для всей цепи:

    Примечание: В T и R T являются напряжение и сопротивление для всей цепи, включая внешнее и внутреннее сопротивление.

    Таким образом, 9 & Omega; резистор получает V = IR = 0.2 x 9 = 1,8 В

    Таким образом, эта ячейка ЭДС на 2 В фактически подает 1,8 В во внешнюю цепь.

    Пример 2:

    Теперь поменяйте местами 9 & Omega; резистор в последнем примере для 1 & Omega; резистор.

    Ответ:

    Найдите V, терминал pd, снова используя тот же метод:

    Теперь ячейка ЭДС 2V подает только 1V во внешнюю цепь !!! Другой 1 В теряется, что приводит к нагреву элемента. Не очень эффективно.

    Примечание: При принятии решения о том, подходит ли элемент для использования в конкретной цепи, необходимо учитывать внутреннее сопротивление. Для максимальной эффективности внешнее сопротивление должно быть намного больше внутреннего сопротивления ячейки. Однако для максимальной мощности, передаваемой во внешнюю цепь, внутреннее сопротивление должно быть равно сопротивлению внешней цепи, хотя эффективность ячейки будет только 50%.

    Источники питания с низким напряжением и более высокими токами, например автомобильный аккумулятор, должны иметь низкое внутреннее сопротивление, как показано выше.Источники питания высокого напряжения, вырабатывающие тысячи вольт, должны иметь чрезвычайно высокое внутреннее сопротивление, чтобы ограничить ток, который может протекать в случае случайного короткого замыкания.

    Поскольку V = E — Ir, если вы построите график зависимости pd, V, от тока I, градиент графика будет равен внутреннему сопротивлению ячейки.

    alexxlab

    Добавить комментарий

    Ваш адрес email не будет опубликован. Обязательные поля помечены *